Среда , 24 Апрель 2024

Загадки с подвохом с ответом: Загадки с подвохом и на логику

Содержание

Загадки с подвохом и на логику

Во Франции и Германии это означает ноль, в Японии — деньги, в Тунисе — угроза смерти, в Сирии — разрыв отношений. А в США?
Ответ: ОК (знак, который можно показать пальцами)
1658

В каком случае 10 взрослых и 10 детей, забравшись под один зонтик, не намокнут?
Ответ: Когда нет дождя
2421

Сидит человек. А вы не можете сесть на его место, даже если он встанет и уйдёт. Где он сидит?
Ответ: На ваших коленях
3631

Когда человек бывает в комнате без головы?
Ответ: Когда он высовывает голову через окно на улицу
5357

Что чем больше ешь, тем больше остаётся?
Ответ: Орех, скорлупа
5342

Что можно видеть с закрытыми глазами?
Ответ: Сон
5718

Не было и не будет, а названье было и будет.
Ответ: Птичье молоко
5251

У трёх шоферов был брат Андрей, а у Андрея братьев не было. Могло ли это быть?
Ответ: Да, если шофёры были женщинами
3800

Что может в одно и то же время

Висеть и стоять, стоять и ходить,
Ходить и лежать, лежать и врать?
Ответ: Часы
3866

Какая одежда состоит из ста частей без единого шва?
Ответ: Перо, пух
4673

Кто без чего не обойдётся?
Ответ: Человек без имени
5203

На какой вопрос никто никогда не ответит «да»?
Ответ: Спящий на вопрос: «Вы спите?»
Безруков Александр, Рассказово
1642

По какому пути никто не ходил и не ездил?
Ответ: По млечному
3711

Что за трава, которую даже слепые знают?
Ответ: Крапива
4341

Какой месяц короче всех?
Ответ: Май (три буквы)
4107

Корова — 2, кошка — 3, овца — 2, кукушка — 4, петух — 8, собака — 3, осёл — ?
Ответ: 2 (иа)
5443

На что больше всего походит половина апельсина?
Ответ: На другую половину апельсина
3800

В каком месяце люди едят меньше всего?
Ответ: В феврале
4317

Если бросить в Чёрное море красный камень, то какой он будет?

Ответ: Мокрый
3291

Какой огонь не жжёт?
Ответ: Нарисованный
4660

Чем больше из неё берёшь, тем больше она становится.
Ответ: Яма
3157

Шесть ног, а бежит не быстрее, чем на четырёх.
Ответ: Всадник на коне
3471

Какая еда, хоть вари её в пяти пудах соли, никогда не будет солёной?
Ответ: Яйцо
2757

Что вчера было «завтра», а завтра будет «вчера»?
Ответ: Сегодня
3542

Можно ли в решете воды принести?
Ответ: Можно, когда она замёрзнет
3858

Сколько горошин может войти в обыкновенный стакан?
Ответ: Сами они войти не могут
3022

Может ли страус назвать себя птицей?
Ответ: Нет, он не умеет говорить
3498

Когда руки бывают тремя местоимениями?
Ответ: Когда они вы-мы-ты
3299

Какое слово всегда звучит неверно?
Ответ: Слово «неверно»
3562

На какое дерево садится ворона во время дождя?
Ответ: На мокрое
2710

Сын моего отца, а мне не брат. Кто это?
Ответ: Я сам
3791

Когда в пустом кармане есть что-нибудь?
Ответ: Когда в кармане дырка
3222

Из какой посуды нельзя ничего поесть?
Ответ: Из пустой
2960

Шёл дождь. Ехал автобус. В автобусе все спали и только водитель не спал. Какой номер автобуса и как звали водителя?
Ответ: номер автобуса мокрый, водителя звали Толька ~ «только водитель»
3629

А и б сидели на трубе, а — упало, б — пропало. Что осталось на трубе?
Ответ: Осталась и
2897

Какие часы показывают правильное время только два раза в сутки?
Ответ: Сломанные механические
2629

В названии какого города имя одного мальчика и имя ста девочек?
Ответ: Севастополь
2749

Когда мужчину называют женским именем?

Ответ: Когда он спит. Соня
3163

У сороки две, у человека одна, у медведя ни одной.
Ответ: Буква «о»
2860

Что не может поместиться в самую большую кастрюлю?
Ответ: Крышка от этой кастрюли
Приставка Мария
1253

Сколько яиц можно съесть натощак?
Ответ: Одно, остальные уже не натощак
3202

Чем заканчивается день и ночь?
Ответ: Буквой «ь»
2979

Страницы: 1 2 … 9 10 11

Если загадок с подвохом вам оказалось мало, посмотрите загадки схожей тематики: прикольные и смешные загадки.
На тему загадок с подвохом для вас может быть интересной наша статья «Как нарисовать 1000 человечков за одну минуту?»



Загадки обманки для детей с ответами

Загадки обманки — умопомрачительно весёлые загадки с неожиданными ответами для детей и их родителей.

В чем суть загадок обманок?

Суть загадок обманок для детей в том, что ответ, который хочется дать в рифму стишка, не является правильным.

Как правильно разгадывать загадки обманки?

Загадки обманки с ответами заканчиваются словом в рифму, но ответ должен быть совершенно не таким. Прочитайте полностью стишок и, вы увидите, что «на автомате» как дети, так и взрослые дают неверный ответ. Нужно совсем немножечко подумать, чтобы догадаться, какое слово является разгадкой на самом деле.

ВНИМАНИЕ: Правильный ответ указан в скобках при нажатии на слово «ответ».

Белым снегом всё одето —
Значит, наступает …

лето (зима)

ОТВЕТ

Ночью каждое оконце
Слабо освещает . ..

солнце (луна)

ОТВЕТ

Друг зверей и друг детей
Добрый доктор …

Бармалей (Айболит)

ОТВЕТ

Кукарекает спросонок
Милый, добрый …

поросёнок (петух)

ОТВЕТ

Высокий, длинноногий,
Летать ему не лень —
На крыше из соломы
Устроился …

олень (аист)

ОТВЕТ

Лишь только свет дневной потух,
заухал в темноте …

петух (филин)

ОТВЕТ

Чик-чирик! Чик-чирик! —
Кто поднял веселый крик?
Эту птицу не пугай!
Расшумелся …

попугай (воробей)

ОТВЕТ

Под деревом четыре льва,
Один ушёл, осталось …

два (три)

ОТВЕТ

Нашёл пять ягодок в траве
И съел одну, осталось . ..

две (четыре)

ОТВЕТ

Мышь считает дырки в сыре:
Три плюс две — всего …

четыре (пять)

ОТВЕТ

Кто взлетит с цветка вот-вот?
Разноцветный …

бегемот (мотылёк)

ОТВЕТ

С пальмы вниз, на пальму снова
Ловко прыгает …

корова (обезьяна)

ОТВЕТ

Простой вопрос для малышей:
Кого боится кот?..

мышей (собак)

ОТВЕТ

Хвост веером, на голове корона.
Нет птицы краше, чем …

ворона (павлин)

ОТВЕТ

Мимо улья проходил
Косолапый …

крокодил (медведь)

ОТВЕТ

В чаще, голову задрав,
воет с голоду …

жираф (волк)

ОТВЕТ

Дочерей и сыновей
учит хрюкать . ..

соловей (кабан)

ОТВЕТ

Кто грызёт на ветке шишку?
Ну, конечно, это …

мишка (белка)

ОТВЕТ

Ква-ква-ква — какая песня!
Что быть может интересней,
Что быть может веселей?
А поет вам …

соловей (лягушка)

ОТВЕТ

Скорей на берег выбегай!
Плывет зубастый …

попугай (крокодил)

ОТВЕТ

Как? Неизвестно до сих пор:
секрет и есть секрет,
зверь этот, словно светофор,
свой изменяет цвет.
В зелёный, жёлтый… Напугай —
и покраснеет …

попугай (хамелеон)

ОТВЕТ

Cъела зайчонка и ловит второго
огненно-рыжая злая …

корова (лиса)

ОТВЕТ

Он пиявок добывал,
Карабасу продавал,
Весь пропах болотной тиной,
Его звали. ..

Буратино (Дуремар)

ОТВЕТ

Он гулял по лесу смело,
Но лиса героя съела.
На прощанье спел бедняжка.
Его звали …

Чебурашка (Колобок)

ОТВЕТ

Много дней он был в пути,
Чтоб жену свою найти,
А помог ему клубок,
Его звали …

Колобок (Иван-Царевич)

ОТВЕТ

И красива, и мила,
Только очень уж мала!
Стройная фигурочка,
А зовут …

Снегурочка (Дюймовочка)

ОТВЕТ

Я красивый, я летаю,
А весной от солнца таю.
Угадайте поскорей,
Кто же это? …

воробей (снег)

ОТВЕТ

Он большой шалун и комик,
У него на крыше домик.
Хвастунишка и зазнайка,
А зовут его …

Незнайка (Карлсон)

ОТВЕТ

Утром рано я встаю,
Молочком всех напою,
Травку я жую за речкой,
А зовусь я как? . ..

овечка (корова)

ОТВЕТ

Жил в бутылке сотни лет,
Наконец, увидел свет,
Бородою он оброс,
Этот добрый …

Дед Мороз (Старик Хоттабыч)

ОТВЕТ

Все меня боятся —
Я могу кусаться,
Я летаю и пищу —
Жертву я себе ищу,
Ночью мне уж не до игр,
Угадали, кто я? …

тигр (комар)

ОТВЕТ

Я лаю и кусаю,
Я дом ваш охраняю,
Всегда смотрю во все глаза,
А как зовут меня? …

коза (собака)

ОТВЕТ

С голубыми волосами
И огромными глазами,
Эта куколка — актриса,
А зовут ее …

Алиса (Мальвина)

ОТВЕТ

И капризна, и упряма,
В детский сад не хочет …

мама (дочка)

ОТВЕТ

На прививки и уколы
Мамы деток водят в . ..

школы (поликлиники)

ОТВЕТ

Математические загадки с подвохом, математические загадки для детей с ответами

Не все так серьезно и скучно в математике, как может показаться на первый взгляд. Предлагаю немножко поиграться с числами, отгадывая веселые математические загадки от Пустунчика. Но не торопись с ответом — в каждом вопросе есть подвох.

34 92 т.

Можешь предложить занимательное математическое соревнование одноклассникам. Поделитесь на две команды (или играйте каждый сам за себя) и начинайте отгадывать на скорость. Интересно, кто из вас заслужит почетное звание математического гения?

Как говорится, Бог в помощь, ветер в паруса. Начали!

1. Ехал как-то мотоциклист в город. По дороге ему повстречались пять легковых автомобилей и грузовик. Сколько машин ехало в город?

2. Было в одной семье два отца и два сына. Сколько это человек?

3. Мальчик купил книгу с оберткой за 11 гривен. За саму книгу он заплатил на 10 гривен больше стоимости обертки. Назовите отдельную цену книги и обертки.

4. Жили-были пять сыновей, и у каждого из них была сестра. Посчитайте, сколько же детей в семье?

5. Два хороших товарища вышли навстречу друг другу: первый из пункта А со скоростью 20 км/ч, а второй из пункта В со скоростью 15 км/ч. Кто из них будет ближе к пункту А, когда друзья встретятся.

6. Одно яйцо варится 4 минуты, сколько времени нужно варить шесть яиц?

7. Есть 4 яблока. Подели их, не разрезая, между тремя друзьями таким образом, чтобы никто из друзей не получил больше чем остальные.

8. Часы с боем отбивают 1 удар за 1 секунду. За какое время они отобьют 12 часов?

Ответы:

1. Как минимум одна.

2. Три человека: дед, отец и сын.

3. Обертка стоила 50 копеек, а книга — 10 гривен 50 копеек.

4. Шесть детей.

5. Они встретятся на одинаковом расстоянии от пункта А.

6. Четыре минуты (или немножко больше).

7. Два яблока даем одному другу, и по одному — двум другим.

8. За 11 секунд.

Вот такая вот арифметика для малышей. Признайся честно, сколько из этих вопросов удалось осилить Тебе?

Заметили орфографическую ошибку? Выделите её мышкой и нажмите Ctrl+Enter

Загадки с подвохом с ответами

Кирпич весит 1 килограмм и еще столько сколько полкирпича.
Сколько весит кирпич?

(Решение: Мы не знаем сколько весит кирпич, пусть он будет х, тогда
х=1+х/2
х-х/2=1
0,5х=1
х=2)
Еще можно рассуждать так: если масса одного кирпича равна 1кг+полкирпича,
то масса двух кирпичей — 2кг и еще кирпич.
Отсюда ясно, что масса кирпича — 2кг.

Сто одежек и все без застежек.

(Сэконд хэнд)

Старый дед, во сто шуб одет, кто его раздевает, тот слезы проливает.

(Бомж)

Сам алый, сахарный, кафтан — зеленый, бархатный.

(Пьяный прапор)

Краса девица сидит в темнице, а коса — на улице.

((Премьер-министр Украины Юлия Тимошенко арес тована российскими властями в районе Тузлинской косы))

Без окон, без дверей — полна горница людей.

(Интернет-чат)

Зимой и летом — одним цветом.

(Черный пиар)

Висит груша, нельзя скушать.

(Тренажер Майка Тайсона)

Назовите слово, в котором 40 гласных.

(Сорока [сорок «а»])

Летел по небу бегемотик, а по земле за ним бежал охотник с ружьем.
Охотник выстрелил, и бегемотик упал на него. Кто жив остался?

(Слоник, потому что он вылетел позже)

Чем отличается публика в цирке от воздушного шарика?

((Воздушный шарик сначала надувают, потом пускают,
а публику сначала впускают, а потом надувают))

Чем человек отличается от паровоза?

(Паровоз сначала свистит, потом трогается,
а человек сначала тронется, а потом ходит и свистит)

Чем отличается молодой холостяк от старого?

(Молодой холостяк прибирается в своем доме,
чтобы пригласить женщину, а старый приглашает в свой дом женщину, чтобы она прибралась)

Что общего между деньгами и гробом?

(И то, и другое сначала заколачивают, а потом спускают)

Два гвоздя упали в воду. Как фамилия грузина?

(Заржавели)

Что это такое: синий, большой, с усами и полностью набит зайцами?

(Троллейбус)

Сколько горошин может войти в один стакан?

(Нисколько, так как горошины не ходят)

Что это такое: висит на стене и пахнет?

(Часы: в них кукушка сдохла)

Почему слоны не летают?

(По воздуху)


Чем лошадь отличается от иголки?

(На иголку сначала сядешь, потом подпрыгнешь,
а на лошадь сначала подпрыгнешь, потом сядешь)

Чем первый этаж отличается от девятого?

(С первого этажа упадешь: «Бух! — А-а!».
А с девятого «А-а! — Бух!»)

Чем кончаются день и ночь?

(Мягким знаком)

Чем отличается трактор от помидора?

(Помидор красный, а в тракторе дверь открывается на ружу)

Что это такое: летит, шуршит, а не шуршавчик?

(Брат шуршавчика)

Что это такое: в небе есть, в земле нет;
у бабы — две, у мужика — ни одной?

(Буква «Б»)

Домашнее животное, на «т» начинается.

(Таракан)

Домашнее животное, на «д» начинается.

(Два таракана)

Домашнее животное, на. «ы» начинается.

(Ышо один таракан)

Что это: сидит на окне, говорит по-французски?

(Француз)

Как правильно говорить: «Не вижу белый желток» или «Не вижу белого желтка»?

(Желток не может быть белым)

Какое слово начинается с трех букв «г»
и заканчи вается тремя буквами «я»?

(«Тригонометрия»)

Что это: стоит черное, на одной ноге?

(Одноногий негр)

Что это: стоит черное, на двух ногах?

(Два одноногих негра)

Что это: стоит черное, на трех ногах?

(Нет, это не три одноногих негра,
а всего лишь рояль)

Что это: черное — на четырех ногах?

(Одноногий негр за роялем)

Почему в Париже девки ходят рыжие?

(По земле)

Один глаз, один рог, но не носорог. Что это?

(Корова из-за угла выглядывает)

Что стоит посреди Земли?

(Буква «м»)

М оскву заводили, во что первый гвоздь колотили?

(В шляпку)

Как правильно сказать: у рыб нет зуб, у рыбов нет зубов или у рыбей нет зубей?

(У рыб есть зубы)

Кто это — маленький, в земле живет, на букву «щ» начинается?

(Щервячок)

Почему поп шляпу покупает?

(Потому что даром не дают)

Чем отличается вода от близнецов?

(Вода — Н2О, а близнецы — О, аж два!)

Маленький, серенький, на слона похож. Кто это?

(Слоник)

Сколько жирафов влезает в «Запорожец»?

(Три: хотя машина и четырехместная,
но одно место предназначено для водителя)

А сколько бегемотов влезает в «Запорожец»?

(Ни одного, потому что все места в «Запорожце» уже заняты жирафами)

Шли три туриста по лесу, и было у них в рюкзаке на троих три пирожка.
В первый день они съели один пирожок.
Во второй день они съели второй пирожок.
На третий день смотрят в рюкзак — пусто!
Куда же делся третий пирожок?

(Они его потеряли)

Двое знакомых ехали в Восточном экспрессе.
У одного была немецкая овчарка, а у другого — оте чественный чемодан.
Как-то раз они поссорились, и в пылу ссоры первый выкинул в окно чемодан,
а второй в отместку выбросил в окно собаку. После этого оба рванули стоп-кран.
Поезд остановился, они выбежали — и видят: к ним несется овчарка, а в зубах у нее… Что?

(Нет, не чемодан! Правильно: в зубах овчарка держит пирожок, который потеряли три туриста в лесу)

От чего утка плавает?

(От берега)


Шел охотник мимо башни с часами.
Достал ружье и выстрелил.
К уда он попал?

(В милицию)

Представьте себе, что из Москвы через центр Земли
насквозь прорыли шахту диаметром один метр и кинули в нее камень диаметром пять сантиметров.
Долетит ли он до противоположного конца шахты?

(Нет, потому что на глубине три метра его съест маленький зелененький камнеглот)

Сколько операций нужно сделать, чтобы поместить жирафа в обыкновенный холодильник?

(Нужно сделать три операции:
открыть дверцу холо дильника, засунуть туда жирафа, закрыть дверцу)

А сколько нужно сделать операций,
чтобы помес тить в холодильник бегемота?

(Нужно сделать четыре операции:
открыть дверцу, вытащить жирафа, засунуть бегемота и закрыть дверцу)

Лев созвал всех зверей на собрание.
Явились все, кроме одного. Что это за зверь?

(Это бегемот. Он же в холодильнике сидит)

Вам нужно пересечь широкую реку, которая кишит крокодилами. Как вы это сделаете?

(Вплавь. Ведь крокодилы-то все на собрании у льва)

Кто быстрее пробежит вокруг Кремля — жираф или бегемот?

(Жираф, потому что бегемот сидит в холодильнике и не может принимать участие в этом соревновании)

Мальчик с девочкой в траве что-то делали на «е».

(Ели землянику)

Что это: маленькое, зелененькое, на глубине три метра сидит и камни глотает?

(Это маленький зелененький камнеглот)

Что это: веселое, красненькое, с пятнышками, из грядки торчит?

(Еще проще: веселый краснопяточный грядкоторчатель)

Сколько требуется негров, чтобы похоронить человека?

(Пять. Четверо несут гроб, а пятый идет впереди с магнитофоном)

Около 40 миллионов человек занимаются ЭТИМ по ночам. Что ЭТО такое?

(Интернет)

С горки ползком, в горку бегом.

(Сопля)

Стоит баба на полу, приоткрыв свою дыру, Что это?

(Печка)

Встанет, до неба достанет. Что это?

(Радуга)

В зубах доска, в глазах тоска. Кто это?
(Мужик провалился в деревенский туалет)
Что нельзя съесть на завтрак?

(Обед и ужин)

Без рук, без ног на бабу скок!

(Коромысло)

Кто под проливным дождем не намочит волосы?

(Лысый)

Одно колесо и тысяча крыльев. Что это?

(Тачка с навозом и мухами)

Что такое: твердое в мягкое вставляется, и шарики рядом болтаются?

(Серьги)

Красный, длинный, 21?

(Трамвай)

Что такое синее золото?

(Любимая жена напилась)

Прыгает ловко и ест морковку? Кто это?

(Легкоатлет на диете)

Две женщины у забора:
одна приклеена, другая пришита…
Что с ними нужно сделать?

(Первую отодрать, вторую — отпороть)

Что возбуждается палочкой Коха?

(1) туберкулез:
2) жена Коха) )

Что объединяет горелый хлеб, утопленника и бере менную женщину?

(Не успели вытащить…)

Что нужно делать, когда видишь зеленого человечка?

(Переходить улицу)

Зеленое, в пятнах, прыгает.

(Десантник)

Что имеет голову, но не имеет мозгов?
(Сыр, лук, чеснок)Один глаз, один рог, но не носорог?

(Корова из-за угла выглядывает)

Какое слово начинается с трех букв «Г» и заканчивается тремя буквами «Я»?

(Тригонометрия)

Мимо башни с часами шёл охотник,
достал ружьё и выстрелил.
Куда он попал?

(В милицию)

В каком случае шесть детей, две собаки, четверо взрослых, забравшись под один зонтик, не намокнут?

(Если не будет дождя)

Эти три телезвезды хорошо известны каждому из нас.
Блондина зовут Степан, шатена зовут Филипп. А как зовут лысого?

(Хрюша)

Что состоит из одного слова,
Но имеет тысячи писем?

(Почта)

Что имеет две головы,
Четыре глаза, шесть ног и хвост?

(Ковбой верхом на коне)

Не лает, не кусается, а так же называется.

(@)

Встанет, до небу достанет.

(Радуга)

Что нельзя съесть на завтрак?

(Обед и ужин)

Кто родился,а еще не умер?

(Тот кот живет)

Кто всегда спит, не снимая туфлей?

(Лошадь)

Когда поп в портках крест выносит?

(Всегда)

Какое изобретение позволяет смотреть сквозь стены?

(Окно)

Мальчик упал с лестницы длиной 15 метров.
Но он не пострадал. Почему?

(Он упал с нижней ступеньки)

За что поп шляпу купил?

(За деньги)

Однажды, когда я завтракал,
Мое обручальное кольцо соскользнуло с пальца
И упало в полную чашку кофе,
Но кольцо не намокло.
Почему?

(В чашке был молотый кофе)

Отчего корова ложится?

(Не умеет сидеть)

Сидят три кошки,
Против каждой кошки —
Две кошки. Много ли всех?

(Три)

Три человека плыли в лодке.
Она перевернулась,
Но только у двоих намокли волосы.
Почему?

(Один из них был лысым)

Сколько букв в алфавите?

(Семь, посчитайте — А-Л-Ф-А-В-И-Т)


Как называется человек,
У которого не все пальцы на одной руке?

(Нормальный. У вас же пальцы на обеих руках)

Какое лучшее средство от перхоти?

(Облысение)

Первый человек является хозяином драгоценных камней,
Второй человек является хозяином любви,
Третий человек является хозяином лопаты,
Четвертый человек является хозяином большой палки.
Кто они?

(Короли в колоде карт)

Что это — большой, как слон,
Но вообще нисколько не весит?

(Тень слона)

Кто под проливным дождем не намочит волосы?

(Лысый)

В пустыне лежит мертвый мужчина. За плечами мешок, на поясе фляга с водой.
На многие километры вокруг нет ни единой живой души.
От чего умер человек и что в его мешке?

(Человек умер от удара об землю,
а в мешке — парашют, который не раскрылся)

В 12-этажном доме есть лифт.
На первом этаже живет всего 2 человека,
от этажа к этажу количество жильцов увеличивается вдвое.
Какая кнопка в лифте этого дома нажимается чаще других?

(Независимо от распределения жильцов по этажам, кнопка «1»)

Маленькое, желтенькое под кроватью лежит, на «З» начинается.

(10 рублевая монета. Почему на «З»? Закатилась…)

Что это такое: летит и блестит?

(Комар с золотым зубом)

Что у женщины на теле, у еврея на уме,
Применяется в хоккее и на шахматной доске?

(Комбинация)

На какой вопрос никто никогда не ответит «да» несоврав?

(Спящий, на вопрос: «Вы спите?»)

Когда человек бывает в комнате без головы?

(Когда высовывает ее из окна на улицу)

Когда козе исполнится семь лет, что будет дальше?

(Пойдет восьмой)

К берегу реки подходят двое мужчин.
У берега одна лодка, которая может выдержать только одного.
Оба мужчины переплыли на противоположный берег на лодке.
Как они это сделали?

(Они подошли к разным берегам)

Как много кубиков можно
Положить в пустую корзину?

(Один. После корзина будет не пуста)

По чему, когда захочешь спать, идешь на кровать?

(По полу)

Что такое: 90/60/90?

(Скорость при гаишнике)

Среднее арифметическое между велосипедом и мотоциклом?

(Мопед)

Что такое черное, когда она чистая,
И белая, когда она грязная?

(Школьная доска)

Висит на стене, зеленое и пищит.

(Селедка. Висит на стене потому,
что я ее туда повесил, зеленая потому, что я ее покрасил,
а пищит, чтобы никто не догадался)

Из какой посуды нельзя ничего поесть?

(Из пустой)

У отца Мэри пять дочерей. Четырёх из них зовут Чече, Чочо, Чуча, Чичя. Как зовут пятую дочь?

(Мэри)


У меня нет костей и нет ног,
Но если меня положить тепло,
Я скоро начну ходить. Кто я?

(Яйцо)

Какое слово всегда звучит неверно?

(Слово «неверно»)

Почему поп шляпу покупает?

(Потому что даром не дают)

Почему собака бегает?

(По земле)

Сколько горошин может войти в стакан?

(0, горошины не ходят)

В небе есть,на земле нет,
У бабы два,а у мужика не одной!

(Буква «Б»)

Сколько фруктов можно съесть натощак?

(Только один фрукт, следующие будут уже не натощак)

В лесу заяц. Пошел дождь. Вопрос: под каким деревом заяц спрячется?

(Под мокрым)

Ехал купец ел солёный огурец.
Одну половинку съел кому вторая досталась?

(Алёне)

Мужик каждый день ездит на лифте с 11 этажа на 1,
а когда возвращается с работы то едет с 1 на 7, а потом пешком. Почему?

(Потому что он карлик и не может дотянуться до 11 кнопки)

👍Загадки с подвохом. Ответом будет ребус!🤔 | Семен Семеныч

Подпишись

на канал! Что бы не пропустить все самое интересное!!!

Здравствуйте! Уважаемые посетители канала «Семен Семеныч»

Сегодня мы с вами попробуем необычный формат загадок.
Ответом может послужить и сама загадка или же ребус.

И то и то является верным решением.

Небольшое условие. Для начала разгадывания нажмите на 👍👍👍👍👍!

🤔Загадка номер один

Итак поехали… Буду давать вам небольшие подсказки, спойлера нет, так что если вы не хотите подсказку, переходите сразу к картинкам.

Ответом послужит ягода та, что для вина французам служит она

Загадка про напиток или явление

Загадка про напиток или явление

Ответ-ребус😁

Ответ за загадку-ребус

Ответ за загадку-ребус

🤔Загадка номер два

Я уверен, что вы справились с решением загадки номер 1. Приступим к загадке номер два!

Будьте внимательней к этой загадке, ответ не простой ищите вы в тексте, рыбак вам подскажет, что ест на обед он.

Загадка про тюрьму и заключенного

Загадка про тюрьму и заключенного

Ребус-ответ😉

Ребус-ответ на загадку про тюрьму и заключенного

Ребус-ответ на загадку про тюрьму и заключенного

🤔Загадка номер три

И так у вас на пути загадка номер три!

Подумайте и скажите ответ, а каком же есть слове, сто раз нет.

Загадка с подвохом про число 100

Загадка с подвохом про число 100

Ребус-ответ😉

Ответ ребус к загадке 3

Ответ ребус к загадке 3

🤔Загадка номер четыре

Прошли вы уже загадки три, четвертая стоит у вас на пути.

Дерево наше, везде тут растет, для мачт корабельных наверно идет.

Задача с подвохом про дерево

Задача с подвохом про дерево

Ребус-ответ😉

Ребус ответ на загадку 4

Ребус ответ на загадку 4

😉Загадка номер пять

Мы плавно уже к концу подошли, загадку последнюю сейчас ты реши!

Не видеть загадку еще нам сто лет, с войною тут связан загадки ответ!

Загадка с подвохом. Про число и век.

Загадка с подвохом. Про число и век.

Ребус-ответ😉

Ответ к загадке про век и число

Ответ к загадке про век и число

Источник загадок : https://zagadki.info/zag/slova.html

Понравилось? Не забудьте поставить лайк и подписаться!

Читайте также другие статьи канала!

Проверь себя на внимательность! Найдёте все загаданные слова?

Задачи на вашу сообразительность и внимательность

Посмотрите внимательно и угадайте кто вор? Загадка на внимательность!

Оставьте свой комментарий как вам такой формат загадок?
Нажми поделиться в своих соц. сетях стрелка⬇⬇⬇

Смешные логические загадки с ответами с подвохом и шуточные

Загадки для взрослых — с подвохом: лучшая подборка

Загадки с подвохом прекрасный способ повеселиться. Правда помните, что подобный тип загадок для взрослых часто имеет слегка пошлый уклон, а потому выбирать их для развлечения нужно только в том случае, если планируется встреча близких друзей.


Загадки с подвохом для взрослых с ответами

Загадки для взрослых — с подвохом:

  1. Самый известный полупроводник. (Ответ — Сусанин.)
  2. Что нужно сделать, если вы сели в машину, а ноги до педалей не достают? (Ответ — можно попробовать пересесть на водительское место.)
  3. А что нужно сделать, если вы сели на водительское кресло, а ноги до педалей по-прежнему не достают? (Ответ — надо для начала повернуться лицом к рулю.)
  4. Что нужно делать, когда увидишь зеленого человечка? (Ответ — быстро переходить улицу, пока горит зеленый цвет светофора. )
  5. Может ли страус назвать себя птицей? (Ответ — мог бы, если бы умел разговаривать.)
  6. Вы сидите в самолете, перед вами находится лошадь, сзади автомобиль. Где вы находитесь? (Ответ — в парке отдыха, на каруселях.)
  7. В каком случае шесть детей, две собаки, четверо взрослых, забравшись под один зонтик, не намокнут? (Ответ — все очень просто, если на улице не будет дождика.
  8. Имя первой женщины в мире, идеально освоившей летательный аппарат. (Ответ — Баба Яга.)
  9. Эти три телезвезды хорошо известны каждому из нас. Блондина зовут Степан, шатена зовут Филипп. А как зовут лысого? (Ответ — всем известный Хрюша.)
  10. В корзине имеется 5 грибов. Как разделить грибы между пятью грибниками, чтобы каждому досталось поровну, и один гриб остался бы в корзине? (Ответ — 4 грибника получат по одному грибу, а пятый получит корзину с одним грибом.)
  11. Два мальчика играли в шашки. Каждый из них сыграл пять партий, и каждый выиграл такое же количество игр. Как такое могло произойти? (Ответ — просто они играли с разными противниками.)

Загадки про взрослых с ответами

Далее предлагаем вашему вниманию подборку загадок про взрослых людей, и связанные с ними жизненными ситуациями. Ответы, конечно, прилагаются.

Загадки про взрослых с ответами:

  1. Что делает сторож, когда у него на голове сидит воробей? (Ответ — сладко спит.)
  2. После тушения пожара, в лесу был найден труп аквалангиста. До ближайшего водоема более 50 км. Как он там оказался? (Ответ — аквалангист искал рыбок в водоеме, пожарный вертолет его случайно зачерпнул и сбросил на очаг пожара в лесу.)
  3. В пустыне лежит мертвый мужчина. За плечами мешок, на поясе фляга с водой. На многие километры вокруг нет ни единой живой души. От чего умер человек и что в его мешке? (Ответ — человек умер от удара об землю, а в мешке — парашют, который не раскрылся. )
  4. Кто под проливным дождем не намочит волосы? (Ответ — человек с зонтом, или лысый человек.)
  5. Когда человек бывает в комнате без головы? (Ответ — когда высовывает ее из окна или дверей.)
  6. Советским дворникам взяли и укоротили метлы. Зачем? (Ответ — советские дворники любили постоять, опираясь на метлу.)
  7. Косоглазый, маленький, в белой шубке, в валенках? (Ответ — Дед Мороз родом из Чукотки.)
  8. Мальчик упал с 4 ступенек и сломал ногу. Сколько ног сломает мальчик, если упадет с 40 ступенек? (Ответ — всего одну, вторая у него уже сломана.)
  9. Что такое, висит на стене и плачет? (Ответ — альпинист.)
  10. Может ли женщина сделать мужчину миллионером? (Ответ — может, если он миллиардер.)

Загадки для взрослых — веселые


Загадка с ответом

Если ваша цель веселье, то подборка веселых загадок для взрослых поможет сделать ваше времяпровождение максимально легким и непринужденным.

Загадки для взрослых — веселые:

  1. Почему загадки опасны? (Ответ — потому что люди ломают над ними голову.)
  2. Что такое бизнес? (Ответ — это способ отбирать деньги без помощи грубой силы.)
  3. Что будет, если скрестить клопов со светлячками? (Ответ — густая сеть электрификации по всей стране.)
  4. Что такое — 90/60/90? (Ответ — скорость при гаишнике.)
  5. Почему у гориллы большие ноздри? (Ответ — потому что у нее толстые пальцы.)

Шуточные загадки настроение каждого праздника

Удачно подобранные конкурсы и головоломки поднимают настроение на любом торжестве. Оригинальные, сложные, прикольные и смешные загадки станут отменным развлечением для каждого гостя, не оставив равнодушным никого. Они же помогают выполнить ряд следующих задач:

  • заполнение паузы между праздничными играми, танцами и смешными конкурсными программами. Продолжительность занимательного мероприятия незначительная, однако, ее достаточно для расслабления и эмоциональной разрядки гостей;
  • создание веселой праздничной викторины. Для большей заинтересованности гостей можно запастись деньгами от «банка приколов», которые станут награждением за правильное решение. Чем больше человек даст правильных отгадок на смешные загадки, тем больше насобирает денег. Ну а на накопленные средства можно будет «купить» любое желание, предварительно заготовленное ведущим программы. Это вовсе не обязательно будет материальный приз. Как вариант, можно пожелать танец тамады, к примеру, на столе. Или еще что-нибудь в таком роде. Гости моментально подключаться и буквально замельтешат ответами с надеждой на победу;
  • не менее интригующим развлекательным вариантом станут прикольные загадки и для командного соревнования. Для решения задачи потребуется две команды игроков, которые получат шуточные загадки в виде списка на листочке. Победит команда, давшая большее число правильных ответов.

Когда праздник будет в самом разгаре, а гости веселые, время использовать усложненные загадки шутки, отгадывание которых одарит торжество особой располагающей атмосферой. Таким образом, каждый проявит логику и эрудицию, блеснув так называемыми «познаниями» окружающей среды.

Отгадывание смешных загадок: хитрые ходы

Чтобы быстро и правильно решать смешные головоломки и отгадывать загадки на логику, не прибегая к помощи знакомых, друзей и родных, сделаем несколько простых тренировочных ходов. Сподручным материалом станут незатейливые головоломки, отгадки на которые уже знакомы. Вооружившись наглядным «пособием», приступаем к практике.

  1. Руководствуясь ответом, разбираемся в принципе составления. Частым обманным маневром считается вопрос человеку о том, что, согласно его мнения, ему не знакомо. Вопрошаемый приходит в замешательство, тем самым отдаляясь от находящегося на поверхности ответа.
  2. Следующим шагом будет разбор загадки на две главные части. Каждую из них нужно поэтапно осмыслить и попытаться дать ответ. Довольно распространенным считается вопрос о личности, у которой две ноги или три. Понятное дело, что смешная загадка о трех ногах с приколом, поэтому задействуем логическое мышление. Не исключено, что здесь предполагаются пошлые отгадки.
  3. Далее внимательно смотрим на действие в загадке. Если неизвестный предмет, допустим, ходит, значит этим глаголом и руководствуемся в ответе. Не факт, конечно, что ходит он естественным образом, возможно, кто-то заставляет его это сделать.
  4. После выявления второстепенной информации обозначаем время события. Логично действия, начинающиеся с утра и заканчивающиеся вечером, заставляют предмет ходить. Здесь применяется переносное значение. Если же центром времени обозначается день, то возможно применение цифры «12». Одним словом, мыслите с широким кругозором.
  5. Заключительным этапом изучения считается сопоставление полученных отгадок на первые вопросы. Определенно веселые головоломки требуют нестандартной логики, поэтому в решении подразумеваются короткие отгадки рифмованной формы или же двухзначные варианты нестандартного плана.

Влиятельным фактором при разгадывании считается определение вида смешные загадки. Так, незамысловатые головоломки подразумевают стандартный вопрос, а вот смешные загадки, требующие применения креативных математических способностей, часто содержат хитроумный ответ.

Обратите внимание! Ассоциации, логичность которых считается само собой разумеющейся, приемлемой, способны сбить человека с толку. А правильный ответ на смешные загадки, «лежащий» на поверхности, плавно исчезнет с поля зрения. Поэтому, вооружившись хитрым подходом, не теряйте из вида даже самые незначительные, на первый взгляд, мелочи, ведь именно они «сигналят» и направляют на нужное решение.

Загадки для взрослых — логические


Логические загадки для взрослых

Если среди ваших друзей и близких есть люди, любящие не совсем банальные загадки, то следующая подборка вам точно понадобится.

Загадки для взрослых — логические:

  1. Как далеко в глубь леса может забежать заяц? (Ответ — до середины леса, так как далее он уже будет выбираться из леса. )
  2. Сколько программистов нужно, чтобы закрутить лампочку? (Ответ — ни одного. Это аппаратная проблема, программисты их не решают.)
  3. Какое колесо не крутится при правом развороте? (Ответ — запасное.)
  4. По чему, когда захочешь спать, идешь на кровать? (Ответ — по полу.)
  5. Что надо сделать, чтобы четыре парня остались в одном сапоге? (Ответ — снять с каждого по сапогу.)
  6. Каких камней в море нет? (Ответ — сухих.)
  7. На сучке сидит ворона. Что надо сделать, чтобы отпилить сук, не потревожив ворону? (Ответ — дождаться пока она улетит.)
  8. На каком языке говорят молча? (Ответ — на языке жестов.)
  9. Вы опередили лыжника, который находился на второй позиции. Какое место вы теперь занимаете? (Ответ — второе, вы заняли его место.)
  10. У отца Мэри есть 5 дочерей: Николь, Агата, Эжени, Лаура. Как зовут 5 дочь? (Ответ — девочку зовут Мери. )
  11. Электричка едет на восток со скоростью 80 км/час. В какую сторону летит дым во время движения? (Ответ — у электрички не может быть дыма. Она передвигается благодаря электричеству.)
  12. Вы вошли в темную комнату взяв коробок спичек, на столе стоит свечка, керосиновая лампа и газовая плита. Что вы зажжете первым? (Ответ — чтобы стало светло, придется зажечь спичку.)
  13. Есть ли где такое поле, где конь через коня прыгает? (Ответ — шахматное.)
  14. Ваши друзья и знакомые используют это чаще чем вы, но это принадлежит вам.
    О чем идет речь? (Ответ — ваше имя.)
  15. Шла женщина в Москву, а навстречу – три мужика. У каждого по мешку, в каждом мешке по кошке. Сколько существ направлялось в Москву? (Ответ — только женщина шла в Москву. Мужчины уже возвращались обратно.)

Загадки для взрослых — прикольные


Прикольные загадки для взрослых

Ну и еще немного загадок для веселой и шумной компании.

Загадки для взрослых — прикольные:

  1. Что такое — падают с ветки золотые монетки? (Ответ — обычное явление в стране дураков.)
  2. Что такое — ходит по стене и играет? (Ответ — муха с плеером в ушах.)
  3. Идет ежик лысый — сколько ему лет? (Ответ — 18, его в армию забирают.)
  4. Чем отличается женское общежитие от мужского? (Ответ — в женском общежитии посуду моют после еды, а в мужском – перед.)
  5. Бывает маленьким, большим. Железо очень дружит с ним. С ним и незрячий непременно найдет иголку в стоге сена. (Ответ — магнит.)
  6. Что такое диетическое яйцо? (Ответ — это яйцо, снесенное курицей, находящейся на диете.)
  7. О чем думал Мюнхгаузен, когда летел на ядре? (Ответ — вот это в боулинге поиграл.)
  8. Кто получит бесплатный сыр в мышеловке? (Ответ — вторая мышка.)
  9. Какая самая не любимая фраза сапера? (Ответ — одна нога здесь, другая – там.)
  10. Может ли собака получить инфаркт? (Ответ — может, если ей создать человеческие условия.)

Загадки для взрослых — сложные


Сложные загадки для взрослых

Если в вашем доме бывают люди, которым по душе более сложные ребусы, тогда попробуйте их удивить и предложите разгадывать сложные загадки.

Загадки для взрослых — сложные:

  1. Один оборот вокруг Земли спутник делает за 1 час 40 минут, а другой — за 100 минут. Как это может быть? (Ответ — 100 минут – это и есть 1 час 40 минут.)
  2. Что не имеет длины, глубины, ширины, высоты, а можно измерить? (Ответ — время, температура)
  3. Если в 12 часов ночи идет дождь, то можно ли ожидать, что через 72 часа будет солнечная погода? (Ответ — нет, так как через 72 часа будет ночь.)
  4. Что может путешествовать по миру, оставаясь в одном и том же углу? (Ответ — почтовая марка на конверте.)
  5. Какой возраст у ребенка самый хороший? (Ответ — когда вы уже не водите его за руку, а он еще не водит вас за нос.)
  6. Что это, две руки, два крыла, два хвоста, три головы, три туловища и восемь ног? (Ответ — всадник, держащий в руках курицу.)
  7. На дереве сидело 10 птиц. Пришел охотник и подстрелил одну птицу. Сколько птиц осталось на дереве? (Ответ — ни одной, остальные испугались выстрела и улетели.)
  8. Товар сначала подорожал на 10%, а потом подешевел на 10%. Какова его стоимость сейчас относительно первоначальной? (Ответ — 99%: после подорожания к 100% прибавили 10% — получилось 110%; 10% от 110% = 11%; затем от 110% отнимаем 11% и получаем 99%.)
  9. Вы проехали на машине две трети пути. В начале пути бензобак машины был полон, а сейчас он заполнен на одну четверть. Хватит ли бензина до конца пути, при аналогичном расходе? (Ответ — нет, так как 1/4 <1/3.)
  10. Один глухонемой человек зашел в магазин канцтоваров, чтобы купить точилку для карандашей. Он воткнул себе палец в левое ухо и кулаком другой руки сделал вращательное движение около своего правого уха. Продавец сразу понял, о чем его просят. Потом в тот же магазин вошел слепой человек. Как он объяснил продавцу, что хочет купить ножницы? (Ответ — просто сказал, он ведь слепой, а не немой.)
  11. В подъезд одновременно зашли 2 человека. У одного квартира на 3-м этаже, у другого — на 9-м. Во сколько раз первый человек доедет быстрее второго? Примечание: они одновременно нажали кнопки в 2 лифтах, движущихся с одинаковой скоростью. (Ответ — обычный ответ: в 3 раза. Правильный ответ: в 4 раза. Лифты обычно ездят с 1-го этажа. Первый проедет 3-1=2 этажа, а второй 9-1=8 этажей, т.е. в 4 раза больше.)
  12. Один студент рассказывает другому: «Вчера баскетбольная команда нашего колледжа выиграла встречу по баскетболу со счетом 76:40. При этом ни один баскетболист в этом матче не забросил ни одного мяча. (Ответ — играли женские команды.)
  13. Мужчина заходит в магазин, покупает колбасу и просит ее нарезать, но не поперек, а вдоль. Продавщица спрашивает: «Вы пожарник?» — «Да». Как она догадалась? (Ответ — мужчина был в форме.)
  14. На одной одесской улице было три портняжные мастерские. Первый портной рекламировал себя так: «Лучшая мастерская в Одессе!» Второй — «Лучшая мастерская в мире!» Третий «переплюнул» их обоих. (Ответ — «Лучшая мастерская на этой улице!»)
  15. Если пять кошек ловят пять мышей за пять минут, то сколько времени нужно одной кошке, чтобы поймать одну мышку? (Ответ — пять минут.)

Загадки – обманки с подвохом про всё на свете

Утром рано по утру,
Всех разбудит … ©
(Неверный ответ – Кенгуру; Верный ответ – Петух, будильник)

***

Утром плотный завтрак нужен,
А вот днем полезен … ©
(Неверный ответ – Ужин; Верный ответ – Обед)

***

И капризна, и упряма,
В детский сад не хочет …
(Неверный ответ – Мама; Верный ответ – Дочка)

***

Взяв за ручку по утрам
Ведут до школы папы …
(Неверный ответ – Мам; Верный ответ – Детей)

***

Утром в каждом доме драма –
Кашу есть не хочет …
(Неверный ответ – Мама; Верный ответ – Ребенок)

***

На базар идут старушки
Покупать себе …
(Неверный ответ – Игрушки; Верный ответ – Продукты)

***

На прививки и уколы
Мамы деток водят в …
(Неверный ответ – Школу; Верный ответ – Поликлиннику)

***

В ползунках всегда одет
Спит в саду с пустышкой …
(Неверный ответ – Дед; Верный ответ – Малыш)

***

День рожденья на носу —
Испекли мы…
(Неверный ответ – Колбасу; Верный ответ – Торт)

***

На обед сыночку Ване
Мама варит суп в…
(Неверный ответ – Стакане; Верный ответ – Кастрюле)

***

Попросила мама Юлю
Ей чайку налить в…
(Неверный ответ – Кастрюлю; Верный ответ – Чашку)

***

Вся хрипит, чихает Лада:
Съела много…
(Неверный ответ – Шоколада; Верный ответ – Мороженого)

***

Холоднющее местечко
В доме нашем это …
(Неверный ответ – Печка; Верный ответ – Холодильник)

***

Просит бабушка Аркашу
Из редиски скушать …
(Неверный ответ – Кашу; Верный ответ – Салат)

***

Говорит нам папа басом:
«Я люблю конфеты с…
(Неверный ответ – Мясом; Верный ответ – Шоколадом, кремом…)

***

Чинят крыши, мебель, рамы,
На рыбалку ходят …
(Неверный ответ – Мамы; Верный ответ – Папы)

***

Я рисую акварелью,
Папа пилит доски …
(Неверный ответ – Дрелью; Верный ответ – Пилой)

***

Ни к чему весь этот спор —
Резать ткань бери…
(Неверный ответ – Топор; Верный ответ – Ножницы)

***

Куклам платьица, штанишки
Любят шить всегда…
(Неверный ответ – Мальчишки; Верный ответ – Девчонки)

***

Лет, наверно, двести
Петиной…
(Неверный ответ – Невесте; Верный ответ – Черепахе)

***

Красным лаком захотела
Я себе покрасить…
(Неверный ответ – Тело; Верный ответ – Ногти)

***

Гладить майку и трусы,
Берем горячие …
(Неверный ответ – Часы; Верный ответ – Утюг)

***

Всем домам нужна охрана,
Дверь не может быть без…
(Неверный ответ – Крана; Верный ответ – Замка)

***

И в Воронеже, и в Туле,
Дети ночью спят на…
(Неверный ответ – Стуле; Верный ответ – Кровати)

***

Белым снегом всё одето –
Значит, наступает …
(Неверный ответ – Лето; Верный ответ – Зима)

***

Все зимою любят дети,
Ездить на … ©
(Неверный ответ – Велосипеде; Верный ответ – Санках)

Загадки – обманки про профессии

К первоклашкам входит в класс
Лишь бесстрашный…
(Неверный ответ – Водолаз; Верный ответ – Учитель)

***

Черный весь, как-будто грач,
С нашей крыши лезет…
(Неверный ответ – Врач; Верный ответ – Трубочист)

***

Чинит в доме всё что можно
Очень мастерски … ©
(Неверный ответ – Художник; Верный ответ – Ремонтник)

***

Лекарства нам в аптеке
Продаст …
(Неверный ответ – Библиотекарь; Верный ответ – Фармацев)

***

На рояле, пианино
Вальс исполнит …
(Неверный ответ – Балерина; Верный ответ – Пианистка)

***

Ходят по клеточкам конь и ладья,
Ход свой победный готовит …
(Неверный ответ – Судья; Верный ответ – Шахматист)

***

Высадить новые саженцы в ельник
Снова отправится утром наш …
(Неверный ответ – Мельник; Верный ответ – Лесник)

***

Под куполом цирка в опасный полёт
Отправится смелый и сильный …
(Неверный ответ – Пилот; Верный ответ – Воздушный гимнаст)

***

Складки, карманы, оборки и кант –
Платье красивое сшил …
(Неверный ответ – Музыкант; Верный ответ – Портной)

***

Кто пасёт коров, овец?
Ну, конечно, …
(Неверный ответ – Продавец; Верный ответ – Пастух)

***

Булки нам и калачи
Каждый день пекут …
(Неверный ответ – Врачи; Верный ответ – Пекари)

***

Арий, опер сочинитель
Называется …
(Неверный ответ – Учитель; Верный ответ – Композитор)

***

В классе английский слышится говор –
Новую тему даёт детям …
(Неверный ответ – Повар; Верный ответ – Учитель)

Загадки – обманки про буквы, цифры и счет

Мы запомнили легко:
Номер первый – буква …
(Неверный ответ – О; Верный ответ – А)

***

Вы поверьте мне, друзья,
Что вторая буква … ©
(Неверный ответ – Я; Верный ответ – Б)

***

Буква третья, погляди,
Несомненно это … ©
(Неверный ответ – И; Верный ответ – В)

***

Круглой формы голова,
Той же формы буква … ©
(Неверный ответ – А; Верный ответ – О)

***

Завершает буквы все
В алфавите буква … ©
(Неверный ответ – Е; Верный ответ – Я)

***

Объяснял учитель Ире,
Что два больше, чем …
(Неверный ответ – Четыре; Верный ответ – Один)

***

Под деревом четыре льва,
Один ушёл, осталось …
(Неверный ответ – Два; Верный ответ – Три)

***

Ты на птичку посмотри –
Ног у птицы ровно …
(Неверный ответ – Три; Верный ответ – Две)

***

Вышел зайчик погулять,
Лап у зайца ровно …
(Неверный ответ – Пять; Верный ответ – Четыре)

***

Цветов не радуге не счесть,
А всего их ровно … ©
(Неверный ответ – Шесть; Верный ответ – Семь)

***

Нашёл пять ягодок в траве
И съел одну, осталось …
(Неверный ответ – Две; Верный ответ – Четыре)

***

Мышь считает дырки в сыре:
Три плюс две – всего …
(Неверный ответ – Четыре; Верный ответ – Пять)

***

На уроках будешь спать,
За ответ получишь …
(Неверный ответ – Пять; Верный ответ – Два)

Загадки – обманки про животных

Где немытая посуда
Там усач живет не худо,
Очень сложно истребим,
Всюду проползет … ©
(Неверный ответ – Пингвин; Верный ответ – Таракан)

***

Села птичка на дубу,
Завела свое «ку-ку»
Налетела вдруг гроза
Улетела … ©
(Неверный ответ – Стрекоза; Верный ответ – Кукушка)

***

Вдоль по листику ползет,
Бедный листик весь сгрызет.
У нее так много ног…
Кто же это ? … ©
(Неверный ответ – Осьминог; Верный ответ – Гусеница)

***

Жизнь их связана с трудом,
Их горой обычно дом,
Тащат в домик всё подряд
Дружный стройотряд …©
(Неверный ответ – Цыплят; Верный ответ – Муравьев)

***

Он пищит, когда летит!
А укус его зудит.
Пока осень не задула,
Всех кусает нас … ©
(Неверный ответ – Акула; Верный ответ – Комар)

***

В пруду, болоте иль реке
На одной она ноге,
Смотрит тщательно вокруг,
Лягушку хочет съесть … ©
(Неверный ответ – Паук; Верный ответ – Цапля)

***

Всё в нем статно и красиво,
Тело, ноги, хвост и грива,
Скачет словно ураган
По полям бежит … ©
(Неверный ответ – Баран; Верный ответ – Конь)

***

Чистый двор ей слишком скушен,
Ей бы поваляться в лужах,
Съест обед и хрюкнет сладко,
В лужу вновь бежит … ©
(Неверный ответ – Лошадка; Верный ответ – Свинья)

***

Ночью слышен громкий вой,
Он всегда голодный злой,
Чуть в капкан не угодил,
Злой, зубастый … ©
(Неверный ответ – Крокодил; Верный ответ – Волк)

***

Крылья пестрые расправит
И порхает и порхает,
Все любуемся мы ей,
Это крошка … ©
(Неверный ответ – Муравей; Верный ответ – Бабочка)

***

Двор и дом свой охраняет,
Чужаков не подпускает,
У него отличный нюх
В конуре живет … ©
(Неверный ответ – Петух; Верный ответ – Собака)

***

Мягко в джунглях он ступает
Никого не подпускает,
Его грива, как огонь,
Трон зверей возглавил… ©
(Неверный ответ – Конь; Верный ответ – Лев)

***

С пальмы вниз,
На пальму снова
Ловко прыгает…
(Неверный ответ – Корова; Верный ответ – Обезьяна)

***

В доме я ловлю мышей,
Развлекаю малышей.
Голос мой ласкает слух,
Называюсь я…
(Неверный ответ – Петух; Верный ответ – Кошка)

***

Сильный, смелый и игривый
Ходит зверь с косматьй гривой.
Прячьтесь , звери, поскорей,
Ходит грозный …
(Неверный ответ – Воробей; Верный ответ – Лев)

***

Посредине океана
Поднялась струя фонтана.
Без труда и без забот
Жизнь в воде проводит …
(Неверный ответ – Кот; Верный ответ – Кит)

***

Кукарекает спросонок
Этот славный …
(Неверный ответ – Поросенок; Верный ответ – Петух)

***

Зимой в берлоге
Видит сон
Лохматый, косолапый…
(Неверный ответ – Слон; Верный ответ – Медведь)

***

На заборе поутру
Кукарекал…
(Неверный ответ – Кенгуру; Верный ответ – Петух)

***

Над лесом солнца луч потух,
Крадётся царь зверей…
(Неверный ответ – Петух; Верный ответ – Лев)

***

Мама с сумкой скок да скок,
В сумке маленький сынок.
Вот так странная семья,
С сумкой прыгает …
(Неверный ответ – Свинья; Верный ответ – Кенгуру)

***

Кто напуган так лисой,
Длинноухий и косой..
Тут трусишке не до игр!
От лисы несётся…
(Неверный ответ – Тигр; Верный ответ – Заяц)

***

В теплой лужице своей
Громко квакал…
(Неверный ответ – Воробей; Верный ответ – Лягушка)

***

По сосне, как в барабан,
Застучал в лесу…
(Неверный ответ – Баран, кабан; Верный ответ – Дятел)

***

Пищу хоботом берёт
Толстокожий…
(Неверный ответ – Бегемот; Верный ответ – Слон)

***

Он не дружит с ярким светом,
Под землёй зимой и летом?
Носом грядки все изрыл.
Кто же это …
(Неверный ответ – Крокодил; Верный ответ – Крот)

***

Ква-ква-ква – какая песня!
Что быть может интересней,
Что быть может веселей?
Так поёт нам …
(Неверный ответ – Соловей; Верный ответ – Лягушка)

***

Клубком свернулся – ну-ка, тронь!
Со всех сторон колючий…
(Неверный ответ – Конь; Верный ответ – Ёж)

***

Чтоб лягушкой поживиться,
Стала в речку это птица.
На одной ноге весь день
Простоит в воде …
(Неверный ответ – Тюлень; Верный ответ – Цапля)

***

Вы меня узнайте сами –
По пескам хожу с горбами.
Ем побеги саксаула,
Потому что я …
(Неверный ответ – Акула; Верный ответ – Верблюд)

***

Умный, серый и свободный,
Я зимой всегда голодный.
А для зайцев я гроза,
Потому что я …
(Неверный ответ – Коза, лиса; Верный ответ – Волк)

***

Кто любит по ветвям носиться?
Конечно, рыжая…
(Неверный ответ – Лисица; Верный ответ – Белка)

***

Кто жильё нам охраняет?
Посторонний не пройдёт:
Испугается он лая.
На пороге верный …
(Неверный ответ – Кот; Верный ответ – Пёс)

***

Нам неизвестно до сих пор:
В чем есть его секрет,
Зверь, как волшебный светофор.
Свой изменяет цвет.
Зеленый, синий… Угадай,
Теперь он красный …
(Неверный ответ – Попугай; Верный ответ – Хамелеон)

***

У него большие уши,
Сам себя облил, как в душе,
Поднял хобот, затрубил
Толстоногий …
(Неверный ответ – Крокодил; Верный ответ – Слон)

***

Пятнистая, рыжая кошка
На ветках проводит весь день.
Под вечер идёт на охоту
Хитрый и быстрый …
(Неверный ответ – Олень; Верный ответ – Ягуар)

***

На лугу меня пасла
Внучка вместе с бабушкой.
Молочка я припасла,
Вот такая …
(Неверный ответ – Бабочка; Верный ответ – Корова)

***

Она под панцирем живет,
На жизнь свою не злится,
Когда куда-нибудь ползет,
То не спешит …
(Неверный ответ – Тигрица; Верный ответ – Черепаха)

***

Длиннее шеи не найдёшь.
Сорвёт любую ветку…
(Неверный ответ – Ёж; Верный ответ – Жираф)

***

Даешь по косточке в обед
Не отнимешь плошки:
Грозно лают и рычат
Озорные …
(Неверный ответ – Кошки; Верный ответ – Щенки, Собаки)

***

У него постель под елью,
Занесённая метелью.
Спит в берлоге без забот
Косолапый …
(Неверный ответ – Бегемот; Верный ответ – Медведь)

***

Хвостик тоненький крючком.
Роет землю пятачком.
В лужу лезет снова
Толстая …
(Неверный ответ – Корова; Верный ответ – Свинья)

***

Быстрее всех от страха
Несется ….
(Неверный ответ – Черепаха; Верный ответ – Заяц, мышь…)

***

Зверь на носу
Свой носит рог –
Ему не подставляйте бок:
Когда сердит, когда он зол,
Все по пути снесет …
(Неверный ответ – Козел, осел; Верный ответ – Носорог)

***

Любит он овсом питаться,
Дом его зовётся – хлев.
«Иго-го! Хочу на волю!
Отпустите!», – просит …
(Неверный ответ – Лев; Верный ответ – Лошадь, конь)

***

Я нектарчик припасу,
Деткам в соты унесу.
А жужжу я там и тут,
Потому что я …
(Неверный ответ – Верблюд; Верный ответ – Пчела)

Загадки – обманки про сказочных героев

Служить злодею были рады,
Кричат «банана» до упада,
Позитивные ребята,
Это желтые … ©
(Неверный ответ – Цыплята; Верный ответ – Миньоны)

***

Есть спасателей команда,
Всё у них отлично, складно,
Всем помогут, всех спасут,
Даже косточку найдут.
Ну ка мне скажите вслух,
Кто же это … ©
(Неверный ответ – Винни Пух; Верный ответ – Щенячий патруль)

***

Родом кажется из леса,
Днем красавица принцесса
Ночью цвет ее зеленый,
Огр Шрек в нее влюбленный,
И она его подружка
Вся зеленая … ©
(Неверный ответ – Лягушка; Верный ответ – Фиона)

***

Волшебство ее из рук
Заморозит все вокруг,
Сестру свою так любит нежно,
Её имя … ©
(Неверный ответ – Белоснежка; Верный ответ – Эльза Холодное сердце)

***

Она живет среди морей,
Она верна мечте своей,
Прекрасны волосы, улыбка,
Чуть в сети не попалась … ©
(Неверный ответ – Рыбка; Верный ответ – Русалочка Ариэль)

***

Этот сказочный герой
Он свирепый и большой,
Как дыхнет, так станет жарко,
Вон с хвостом летит … ©
(Неверный ответ – Русалка; Верный ответ – Дракон)

***

Черный плащ летучей мыши
Может он летать по крышам,
Он герой среди людей,
Почти бессмертный он … ©
(Неверный ответ – Кащей; Верный ответ – Бэтмен)

***

Паутины, как снаряд,
Красно – синий весь наряд.
Лицо всегда скрывает он,
Очень славный, добрый … ©
(Неверный ответ – Гном, слон; Верный ответ – Человек Паук)

***

Самый яркий из пиратов
Капитан и командир.
По морям плывет корабль,
Любит воду … ©
(Неверный ответ – Мойдодыр; Верный ответ – Капитан Джек Воробей)

***

Коли вдруг сломалось что-то
Это их теперь забота,
С инструментом рюкзачок
Ростом мелкий … ©
(Неверный ответ – Пятачок, паучок; Верный ответ – Фиксик)

***

К нам с луны свалился вдруг,
Необычный новый друг,
Кто же это угадай,
Может это … ©
(Неверный ответ – Попугай; Верный ответ – Лунтик)

***

В Простоквашино он жил
И с Матроскиным дружил.
Простоват он был немножко,
Звали песика … ?
(Неверный ответ – Тотошка; Верный ответ – Шарик)

***

Он один из всех котов,
С мышами в мире жить готов,
Но дружить с ними непросто,
Как зовут кота … ? ©
(Неверный ответ – Матроскин; Верный ответ – Леопольд)

***

Он сыграет на гармошке
У прохожих на дорожке.
Не возьму я как-то в толк,
Заяц это или … ?
(Неверный ответ – Волк; Верный ответ – Крокодил Гена)

***

Все узнает, подглядит,
Всем мешает и вредит.
Ей лишь крыска дорога,
Старушка, хитрая … ?
(Неверный ответ – Яга, Лиса; Верный ответ – Шапокляк)

***

Друг зверей и друг детей
Добрый доктор … ?
(Неверный ответ – Бармалей; Верный ответ – Айболит)

***

Потерял он как-то хвостик,
Но его вернули гости.
Он ворчлив, как старичок
Этот грустный … ?
(Неверный ответ – Пятачок; Верный ответ – Иа)

***

Он большой шалун и комик,
У него на крыше домик.
Хвастунишка и зазнайка,
А зовут его … ?
(Неверный ответ – Незнайка; Верный ответ – Карлсон)

***

Она песенки поёт
И орешки все грызет,
Смотрит вся толпа, дивится,
Это рыжая … ©
(Неверный ответ – Лисица; Верный ответ – Белка)

***

Три могучих молодца,
Днем и ночью без конца,
Защищают жизнь людскую,
Землю русскую родную.
Дружно вместе, как три брата
Вышли в поле … ©
(Неверный ответ – Поросята; Верный ответ – Три богатыря)

***

Много дней он был в пути,
Чтоб жену свою найти,
А помог ему клубок,
Его звали …
(Неверный ответ – Колобок, осьминог; Верный ответ – Иван-Царевич)

***

Бедных кукол бьет и мучит,
Ищет он волшебный ключик.
У него ужасный вид,
С бородою … ?
(Неверный ответ – Айболит; Верный ответ – Карабас-Барабас)

***

Он пиявок добывал,
Карабасу продавал,
Весь пропах болотной тиной,
Его звали … ?
(Неверный ответ – Буратино; Верный ответ – Дуремар)

***

Это что за очень странный
Человечек деревянный?
На земле и под водой
Ищет ключик золотой.
Сунул нос свой в котелок.
Кто же это … ?
(Неверный ответ – Колобок; Верный ответ – Буратино)

***

С голубыми волосами
И огромными глазами,
Эта куколка – актриса,
А зовут ее … ?
(Неверный ответ – Алиса; Верный ответ – Мальвина)

***

В Простоквашино жил он,
Пёс веселый …
(Неверный ответ – Артемон; Верный ответ – Шарик)

***

Знает взрослый и малыш,
Что Матроскин – это …
(Неверный ответ – Мышь; Верный ответ – Кот)

***

Сёстры девочку-бедняжку
Называли замарашкой.
Всё помоет, сварит кашу,
Вся в золе обычно …
(Неверный ответ – Маша; Верный ответ – Золушка)

***

Он гулял по лесу смело,
Но лиса героя съела.
На прощанье спел бедняжка.
Его звали … ?
(Неверный ответ – Чебурашка; Верный ответ – Колобок)

***

Из муки его слепили,
Словно солнышко, приметный.
По тропинке покатился
Кто, скажи … ?
(Неверный ответ – Кащей Бессмертный; Верный ответ – Колобок)

***

И красива, и мила,
Родилась в цветке она,
С ноготок фигурочка,
Зовут её … ?
(Неверный ответ – Снегурочка; Верный ответ – Дюймовочка)

***

Жил в бутылке сотни лет,
Наконец, увидел свет,
Бородою он оброс,
Этот добрый … ?
(Неверный ответ – Дед Мороз; Верный ответ – джин Старик Хоттабыч)

***

В дремучем лесу, на болоте
Вы её непременно найдёте.
Не рыбка она, не лягушка,
Она Водяного подружка.
Стройная фигурка,
Зовут ее … ?
(Неверный ответ – Снегурка; Верный ответ – Кикимора Болотная)

***

Внутри него водица,
С ним не хотят водиться,
А все его подружки –
Пиявки да лягушки!
Водорослями он зарос
Это дедушка … ?
(Неверный ответ – Мороз; Верный ответ – Водяной)

***

Есть еще в лесу один
Очень важный господин.
Охраняет он леса,
Все лесные чудеса,
Весь обросший, в палках, шишках,
Так его рисуют в книжках.
Знает он язык зверей,
А зовут его… ©
(Неверный ответ – Кощей; Верный ответ – Леший)

***

Он живет в глуши лесной,
Отрицательный герой.
Он костями громыхает
И в округе всех пугает.
Это что за старичок?
Очень гадкий …
(Неверный ответ – Пятачок, паучок; Верный ответ – Кощей Бессмертный)

***

Волка оседлал он,
И во весь опор,
Мчался от погони
С царевной …
(Неверный ответ – Черномор; Верный ответ – Иван Царевич)

***

Рассказал про Буратино,
Про Мальвину, про Пьеро,
Про Тортилу, Карабаса
Русский сказочник …
(Неверный ответ – Ш. Перро; Верный ответ – Алексей Толстой)

Загадки это те же стихи, при этом они еще более увлекательны за счет того, что ребенку в финале предстоит догадаться о чем или о ком идет речь. Все дети обожают это, именно поэтому загадки так активно используют на различных праздниках, утренниках, на новогодних и на других детских мероприятиях.

Источники

  • https://heaclub.ru/zagadki-dlya-vzroslyh-luchshaya-podborka-s-otvetami
  • http://zagadki-otvetami.ru/smeshnye-zagadki/
  • https://ot2do7.ru/413-zanimatelnye-zagadki-shutki-s-otvetami.html
  • https://super-positive.ru/zagadki-s-podvohom/
  • https://www.prazdnik-sam.ru/zagadki/obmanki-dlya-detei-s-podvohom.html

Пошлые загадки с непошлыми ответами

В последнее время люди все чаще жалуются, что все происходящее опошляют, переводят темы к сексу. Хотите немного разыграть своих друзей, указать им на то, как они все-таки пошло мыслят? Для этого есть несколько пошлых загадок с непошлыми ответами. Ну, и чтобы даже самые просвещенные сразу не просекли фишку, рассказывайте такие загадки с легкой полуулыбкой на лице.

Интересный факт, практически все загадки взяты с советского журнала «Мурзилка», когда-то очень популярного не только среди молодежи, но и среди детей.

Начнем с первой, довольно интересной:

«Волосатая головка за щеку заходит ловко».

Сразу в голову начинают лезть самые неприличные мысли, однако ответ совсем прост: обычная зубная щетка.

В пошлые загадки с подвохом можно отнести и такую:

«Чтобы спереди погладить, нужно сзади полизать».

В голову лезут уж совсем неприличные ассоциации, не правда ли? Но ответ далек от любой пошлости: почтовая марка.

Чтобы совсем возбудить вашу фантазию:

«Выше колена, но ниже пупка, а дырка такая, что влезет рука»!

Такс, что вы подумали? Это обычный карман брюк, и все сходиться на 100%.

А теперь загадка для тех, кто любит занимать интенсивным спортом и хорошенько пропотеть.

«В две руки беру, между ног сую, пять минут потею, но потом балдею».

А вы любите заниматься подобным увлечением? Более того, именно таким спортом занимается две трети населения Земли. Это велоспорт, а ответ на загадку: велосипед.

А теперь другая загадка:

«Без рук и без ног, на женщину скок».

Кто в вашем окружении так лихо завоевывает любую женщину? Ответ прост: у него просто нет конкурентов, одеяло нужно каждому человеку, не только женщинам.

Ну и совсем опрокинуть вашу фантазию в объятия эротического характера:

«Берешь в руки – растет, проводишь меж грудей – растет, вставляешь в нужное отверстие – перестает расти»?

Вы сразу подумали о чем-то пошлом, о чем-то, что есть у любого мужчины. По сути, вы правы, только это есть в каждом автомобиле, это ремень безопасности.

Классика советских пошлых шуточек:

«Сзади тихо подошел, дважды всунул и пошел».

О чем вы сразу вспомнили? Правильно о своих любимых тапочках.

Одна из самых загадочных шуток того времени. С ней вы точно сможете поприкалываться над друзьями:

«Бывает холодным, бывает горячим, бывает висячим, бывает стоячим».

Дадим еще им небольшую подсказку: слово состоит из трех букв, а посередине буква у. Заинтригованы? Душ, конечно, а вы о чем подумали?

А теперь загадка для пожилого поколения. Вы сможете подшутить над своими бабушками и дедушками, а когда они дадут вам подзатыльник за пошлости – сделайте оскорбленный вид. Вы то, ничего непристойного в виду не имели.

«Если бы не бабушкины лохматушки, мерзли бы дедушкины колотушки».

А ответ прост: варежки.

Поговорим немного и на библейскую тему. Даже если вы не христианин, вы все равно хотя бы примерно понимаете, кто такие Адам и Ева. А теперь ответьте на простой вопрос:

Что у Адама спереди, а у красотки Евы сзади?

Это не просто смешная загадка, но и еще простая проверка вашей логики. Итак, барабанная дробь, внимание, ответ! Буква «А».

Пошутим немного и на другую тему. 100% ваших знакомых ответит неправильно, уверяю вас.

«Вокруг растут волосы, а посредине колбаса».

Даже вы сейчас подумали явно неправильную вещь, связанную с мужчиной. Но нет, это вкусный и полезный овощ – кукуруза.

Мужчины точно узнают кто это, вот только ответят неправильно. Итак:

«На букву «х» начинается, на букву «П» поднимается».

Подсказка: связано с одним инстинктом. Ответ связан с милейшим слоником, у которого хобот поднимается на пищу. А о чем подумали вы?

«У какого молодца утром капает с конца»?

Вообще у 50% планеты, но эта загадка совсем не о пошлом. Был в прошлых веках чудо-чайник, самоваром назывался. Вот он и является ответом.

А теперь интересная загадка, на которую есть несколько ответов:

«Больше половины человечества занимаются ЭТИМ по ночам».

Какое занятие имеется в виду? Первый вариант: сидят в интернете. Второй вариант: едят. Ну и  спят. А о чем можно было еще подумать? Все достаточно невинно.

И еще одна:

«Стоит баба на полу, приоткрыв свою дыру».

Ответ: печка. Раньше были популярны другие печи, которые складывались из кирпича и топились дровами.

«Красная головка стоя работает ловко».

Интересно, а как она работает сидя? Никак, потому что дятлы долбят дерево, в основном стоя, в других позициях им это делать неудобно.

«Возьми его в руки… Сожми его крепко… Он станет упругим и твердым как репка…».

Попробуйте так подшутить над знакомой девушкой и посмотреть на ее реакцию. Если она не дала вам пощечину, то значит, она знает, что вы имеете в виду снежок. Рискованно, зато интересно.

Ну и закончим одной из самых популярных загадок прошлого столетия:

«Что ты смотришь на меня? Раздевайся, я твоя!».

Нет, это не легкодоступная женщина, это кровать.

Подшучивайте над знакомыми и наслаждайтесь их реакцией.

А какие вы знаете пошлые загадки с подвохом, но непошлым нормальным ответом? Напишите, пожалуйста в комментариях.

30+ забавных загадок с ответами для взрослых и подростков!

Этот сборник головоломок для взрослых и детей постарше заставит вас почесать голову в недоумении. Но не волнуйтесь, потому что ответы тоже ниже!

Так что разгадывайте себя глупо и посмотрите, сколько из этих сложных загадок вы сможете разгадать, а затем предложите своим друзьям и семье сделать лучше. В конце концов, здоровая конкуренция — это хорошо, а загадки — отличное место для веселого семейного соревнования; просто убедитесь, что вам весело, и не переусердствуйте с конкуренцией! И помните: это сложные и сложные загадки, так что не расстраивайтесь, если вам не удастся разгадать их все!

Действительно сложные загадки для взрослых и подростков

Проверьте свое логическое мышление с помощью этих сложных загадок для взрослых и подростков.Для каждого из них нажмите «Показать ответ», чтобы открыть решение. Удачи!

1. Что нужно сломать перед использованием?

2. У матери Джимми было трое детей. Первый назывался апрель, второй — май. Как звали третьего?

3. У чего есть голова и хвост, но нет тела?

4. Что начинается с Т, заканчивается на Т и содержит Т?

5. Вы можете держать его, не используя руки или руки. Что это?

6. Он настолько хрупок, что его можно сломать даже при слове названия.Что это?

7. В каком здании больше всего этажей?

8. Что на вкус лучше, чем запах?

9. Что сломается, если не хранить?

10. Он существует уже миллионы лет, но никогда не старше месяца. Что это?

11. Накорми меня, и я буду жить, но дай мне пить, и я умру. Что я?

12. Вы выбрасываете внешнее, съедаете внутреннее, а затем выбрасываете внутреннее. Что это?

13. Что можно сохранить после того, как кому-то дадут?

14.У кого четыре пальца и один большой палец, но нет живого?

15. Что плохо пахнет при жизни и приятно пахнет после смерти?

16. Что нельзя класть в кастрюлю?

17. Я живу в своем маленьком домике совсем одна. Здесь нет ни окон, ни дверей, и если я хочу выйти, мне придется пробить стену. Что я?

18. Кто не может говорить, но всегда отвечает, когда к нему обращаются?

19. Три доктора сказали, что Билл был их братом. Билл сказал, что у него нет братьев.Кто лгал?

Показать ответ

Никто. Доктора были сестрами Билла.

20. Уберите мое первое письмо, затем заберите мое второе письмо. Тогда убери остальные мои письма, но я остаюсь прежним. Что я?

21. Чем больше этого есть, тем меньше вы видите. Что это?

22. Я — ящик, в котором хранятся ключи без замков, но мои ключи могут открыть ваши самые сокровенные чувства. Что я?

23. Два отца и два сына в машине, а в машине всего три человека.Как это возможно?

Показать ответ

Это дед, отец и сын.

24. Что имеет один глаз, но не видит?

25. У меня моря без вод, побережья без песка, города без людей и горы без земли. Что я?

26. Он принадлежит вам, но другие люди используют его чаще, чем вы. Что это?

27. На похоронах своей матери женщина встречает парня, которого не знает. Она думает, что этот парень потрясающий. На самом деле он — мужчина ее мечты, и она почти уверена, что он может быть любовью всей ее жизни.Однако она никогда не спрашивала его имя или номер и впоследствии не могла найти никого, кто бы знал, кто он такой. Через несколько дней девушка убивает собственную сестру. Зачем она это сделала?

Показать ответ

Она решила, поскольку этот мужчина был на похоронах ее матери, что он был другом ее семьи, и она надеется, что он снова появится на похоронах ее сестры.

28. Я — одна четверть, но если вы добавите пять, я стану одной третью. Что я?

Покажите ответ

15 минут. (Четверть часа, но добавьте 5 минут, и вы получите двадцать минут, что составляет одну треть часа)

29.Что замерзает при нагревании?

30. Я все, но никто. Я могу быть где угодно или в никуда. Когда гаснет свет, я исчезаю, а когда я смотрю на себя, мы умножаемся. Что я?

Другие забавные загадки

Если вам понравились эти сложные сложные загадки для взрослых и детей старшего возраста, почему бы не попробовать другие наши замечательные загадки и ответы, а также другие наши головоломки и головоломки (конечно, с решениями)? в том числе:

26 грязных загадок для взрослых

Ответы на эти, казалось бы, грязные загадки для взрослых не совсем такие, какими они кажутся.Попробуйте и посмотрите, какой у вас грязный ум! Настоящие ответы находятся внизу страницы.

Внимание! Учтите, что эти загадки подходят только для взрослых, а не для детей.

Загадки о грязных вещах только для взрослых

1. Что входит в сухое и твердое, а получается мягкое и влажное?

2. Какое четырехбуквенное слово, оканчивающееся на «k», означает то же, что и половой акт?

3. Ваш палец точно входит в него. Вы играете с ним, когда вам скучно. Выйдя замуж, вы навсегда останетесь с одним и тем же.Что это?

4. То, что шесть дюймов в длину, ложится в рот, а если вибрирует, будет веселее?

5. Вы играете с ним ночью в постели. Вам не разрешается возиться с этим произведением. Только очень особенным людям разрешается прикасаться к нему. Что это?

6. Что есть у каждой женщины, начинающееся с буквы «V», и что она может использовать, чтобы получить то, что она хочет?

7. Что поднимается, выпускает нагрузку, а затем снова падает?

8. Что у коровы четыре, а у женщины только два?

9.Иногда я причиняю боль, когда вхожу внутрь. Я заполню твои дыры, если ты попросишь меня. Прошу плюнуть, а не глотать. Что я?

10. Что мужчины хранят в штанах, что иногда отсасывают их партнеры?

11. Арнольд Шварценеггер действительно длинный. «Майкл Дж. Фокс» действительно короткий. Микки Маус не человек. У Мадонны его нет. Что это?

12. Что делает собака, на что вы можете ступить?

13. Что начинается с «p» и заканчивается «orn» и действительно популярно в киноиндустрии?

14.Какое четырехбуквенное слово начинается с «f» и заканчивается на «k», и если вы не можете его найти, вы можете вместо этого использовать свои руки?

15. Это весело, но вы ненавидите знать, что ваши родители тоже это делают. Что это?

16. Какое четырехбуквенное слово заканчивается на «оно» и может быть найдено на дне клетки для птиц?

17. Он есть у каждого мужчины. Некоторые большие, некоторые маленькие. Приятно дуть, но если не осторожно, капает. Что это?

18. Что удлиняется при вытягивании, помещается между грудями, аккуратно входит в отверстие, душит людей при неправильном использовании и лучше всего работает при рывках?

19.Что есть у мужчины, которое начинается с буквы «П» и становится больше, если его правильно стимулировать?

20. Что белого, липкого, липкого, и лучше плюнуть, чем проглотить?

21. Что есть в брюках мужчины такого, чего женщина не хочет видеть в своем лице?

22. Включает кровать. Некоторые люди предпочитают его сверху, а другие — снизу. Что это?

23. Я твердая и волосатая снаружи, но мягкая и влажная внутри. Мое имя начинается с «c», заканчивается на «t», между ними есть «u» и «n».Что я?

24. Что красиво и естественно, но становится длинным и колючим, если не подрезать регулярно?

25. У меня жесткий стержень, и мой наконечник проникает внутрь. Я прихожу с колчаном. Что я?

26. Я сильно вхожу, а выхожу мягко. Я не против, если ты меня взорвешь. Что я?

Другие забавные загадки

Если вам понравились эти грязные загадки для взрослых, ознакомьтесь с нашими другими загадками и головоломками, в том числе этими:

16 бесплатных действительно сложных загадок

Если вы нашли другие наши загадки и головоломки слишком легкими, попробуйте вместо этого этот сборник действительно сложных загадок.Это одни из самых сложных загадок, и они обязательно бросят вызов вашему мозгу и дадут ему хорошую работу!

Они могут показаться невозможными, но не волнуйтесь — хотя они и сложны, на все загадки есть ответы, которые можно решить логически. Посмотрите, как хорошо вы справляетесь с этими суперсложными загадками!

16 Really Hard Ridddles

Вы заблудились в лесу, и путь, по которому вы идете, разветвляется на две части: путь налево и один направо. Один путь приведет вас в безопасное место, а другой приведет к потере навсегда.

На развилке две сестры-близнецы, которые знают, какой путь есть. Сестры идентичны во всем, кроме одной: одна из сестер всегда говорит правду, а другая всегда лжет.

Вы можете задать только один вопрос и не знаете, какая сестра какая. Что вы можете спросить у них, чтобы знать, какой путь выбрать?

Показать ответ

Спросите одну из сестер, по какому пути их сестра посоветует вам пойти.

Допустим, путь левой руки — правильный.Сестра, которая лжет, знает, что их правдивая сестра укажет вам путь левой руки, поэтому, поскольку они всегда лгут, они подскажут вам путь правой руки.

Честная сестра знает, что их лживая сестра подскажет вам правильный путь, и, поскольку они честны, они скажут вам это. Поэтому вам следует идти по пути, противоположному тому, что вам говорят, независимо от того, какая сестра вам говорит.


Как камень в дереве, Я помогу твоим словам пережить тебя. Но если вы толкаете меня, когда я стою, чем больше я двигаюсь, тем меньше я становлюсь.

Что я?


Есть три сундука, в каждом по 100 монет. В одном сундуке 100 золотых монет, в другом 100 серебряных монет, а в третьем поровну 50 золотых и 50 серебряных монет.

Каждый сундук промаркирован, но все они промаркированы неправильно. Вам разрешено взять одну монету только из одного из сундуков, и после него вы должны правильно идентифицировать каждый из трех сундуков.

Что делать?

Показать ответ

Возьмите монету из сундука с надписью 50/50.

Если вы выбираете золотую монету, вы знаете, что сундук должен содержать все золотые монеты (это не может быть сундук 50/50, потому что все сундуки неправильно помечены). Таким образом, сундук с надписью «серебро» должен быть сундуком 50/50, а сундук с надписью «золото» должен быть серебряным.

С другой стороны, если вы выбираете серебряную монету, вы знаете, что сундук должен содержать все серебряные монеты, тот, который помечен серебром, должен быть золотым сундуком, а тот, который помечен золотом, должен быть сундуком 50/50.


Я могу жить только там, где есть свет.Но если на меня будет светить, я умру.

Что я?


Произошла авиакатастрофа, в которой погибли все до единого. Тем не менее, выжило 12 человек. Как?

Показать ответ

12 оставшихся в живых были женаты, а не одиноки.


Поверни меня на бок, и я все. Разрежь меня пополам, и я ничто.

Что я?

Показать ответ

Число 8. На боку оно похоже на символ бесконечности, а если разрезать его пополам, оно выглядит как два нуля.


У вас есть две веревки, каждая из которых сгорит ровно за час, если ее поджечь с одного конца. Веревки не идентичны и не однородны, то есть не обязательно, чтобы половина веревки сгорела за полчаса.

Имея только эти две веревки и способ их зажечь, как вы отмеряете 45 минут?

Показать ответ

Вы можете сделать это следующим образом:

Сначала осветите оба конца одной веревки и только один конец другой веревки.

Это приведет к сгоранию первой веревки через 30 минут (если вы зажгли оба конца, она сгорит вдвое быстрее, как если бы вы зажгли только один конец).

Когда сгорит первая веревка, на второй останется 30 минут (так как каждая веревка сгорит в течение одного часа, если она зажжена с одного конца).

Итак, зажгите другой конец второй веревки, чтобы оба ее конца горели. Эта веревка перегорит еще через 15 минут.

Общее время, прошедшее с этого момента, составит 45 минут (30 + 15 = 45 минут).


У меня есть города, но нет домов, леса, но нет деревьев, вода, но нет рыбы.

Что я?


Вы хотите варить яйцо ровно 15 минут, но у вас есть только 7-минутные песочные часы и 11-минутные песочные часы.

Как добиться желаемого?

Показать ответ

Переверните обе песочные часы, когда начнете варить яйцо.

Через 7 минут закончатся 7-минутные песочные часы. Переверните, чтобы начать заново.

Еще через 4 минуты 11-минутные песочные часы закончатся.

Теперь переверните 7-минутные песочные часы. Теперь это займет еще 4 минуты.

Когда это произойдет, ваше яйцо будет кипеть ровно 15 минут (7 минут + 4 минуты + 4 минуты).


Что прыгает, когда ходит, и что сидит, когда стоит?


Уже поздно и темно. Группа из 4-х друзей с одним факелом между ними подходит к реке.

Единственный путь через эту реку — это старый шаткий мост, на котором одновременно могут находиться только 2 человека, и любому, кто пересекает мост, нужен факел, чтобы безопасно перейти.

Элисон может перейти мост за 1 минуту, Брайан — за 2 минуты, Кэрол — за 5 минут, а Дэйву — за 8 минут, чтобы перейти мост.

Каждый раз, когда 2 человека переходят мост вместе, они могут двигаться только со скоростью более медленного человека. Все должны перейти мост за 15 минут.

Как они могут этого добиться?

Показать ответ

Элисон и Брайан вместе переходят мост. Это займет 2 минуты.

Затем возвращается Элисон с фонариком, что занимает 1 минуту.

Затем Кэрол и Дэйв вместе переходят мост, что занимает 8 минут.

Брайан возвращается с факелом. Это займет еще 2 минуты.

Наконец, Элисон и Брайан снова пересекают мост, занимая еще 2 минуты, а общее время составляет 15 минут.


Если ты съешь меня, мой отправитель съест тебя.

Что я?


Мужчина убегает из дома. Он поворачивает налево и продолжает бежать.

Через некоторое время он снова поворачивает налево и продолжает бежать. Позже он снова поворачивает налево и бежит домой.

Кто был тот человек в маске?

Показать ответ

Ловец (в бейсбольном матче).


В подземелье короля было 3 пленника. У двоих из них было прекрасное зрение, но третий заключенный был полностью слеп.

Однажды король собрал пленников и сказал им: «У меня 5 шляп. 2 шляпы черные и 3 белые. Я завяжу каждому из вас глаза, а затем надену по одной из шляп на каждую из вас. Затем я положу оставшиеся две шляпы в другую комнату. Затем я сниму повязки с ваших глаз. В этот момент вы можете смотреть на своих товарищей по заключению, но не можете с ними разговаривать.Человек, который может сказать мне цвет шляпы на его голове, выйдет на свободу. Однако не оскорбляйте меня, угадывая, потому что, если вы угадаете и ошибетесь, вас казнят ».

Затем на головы заключенных надели головные уборы и сняли повязки с глаз.

Затем король обратился к первому пленнику, зрячему человеку. «Какого цвета шляпа у тебя на голове?» Заключенный внимательно огляделся, но, наконец, не желая рисковать смертью, угадывая, сказал, что не знает.

Затем король заговорил со вторым пленником, другим полностью зрячим человеком. «Какого цвета шляпа на голове?» Этот заключенный также внимательно посмотрел на шляпы на головах двух других заключенных, но в конце концов признал, что он тоже не знает.

В этот момент король повернулся и пошел прочь, полагая, что третий пленник, который был слеп, не мог знать, возможно, ответа на загадку.

Но затем заговорил слепой. «Простите меня, милорд, — сказал он, — но я уверен, что моя шляпа белая.

Удивленный, король обернулся. Он увидел, что слепой прав, и приказал немедленно освободить его.

Откуда слепой узник узнал цвет своей шляпы?

Показать ответ

Если бы первый заключенный увидел две черные шляпы на двух других, он бы знал, что на нем белая шляпа, поскольку изначально было только две черные шляпы. Поскольку он не знал, какого цвета у него была шляпа, это означает, что двое других заключенных должны были быть либо в белых шляпах, либо в белой шляпе, а на другом — в черной.

Второй заключенный узнает об этом, если первый заключенный не может сказать, какого цвета его шляпа, т.е. второй заключенный теперь знает, что либо он, либо слепой заключенный носят белые шляпы, либо один из них носит белое, а другой чернить. Итак, если слепой заключенный был одет в черную шляпу, второй заключенный знал бы, что он должен быть одет в белую шляпу.

Поскольку второй заключенный не мог понять, какого цвета он был одет в шляпу, это говорило слепому заключенному, что он должен быть одет в белую шляпу (если бы слепой заключенный был в черной шляпе, второй заключенный мог бы сделать вывод, что он был в белой шляпе).


Вперед я тяжелый, назад нет.

Что я?


В своем офисе найден убитый мужчина. Подозреваемые — Питер, Джули, Джейсон, Молли и Брайан.

В офисе календарь с числами 6,4,9,10,11, написанными кровью.

Кто убийца?

Показать ответ

Джейсон — убийца.

Цифры обозначают месяцы, а первая буква каждого месяца составляет имя убийцы, например 6-й месяц — июнь, первая буква июня — J, 4-й месяц — апрель, первая буква апреля — а, и так далее.


Сложные загадки

Поздравляю, если вы отгадали эти действительно сложные загадки; Вы, должно быть, настоящий умник и хорошо разбираетесь в играх для мозга!

Решили ли вы их все или нет и хотите еще несколько сложных загадок (или даже более простых), почему бы не проверить и другие наши бесплатные загадки и головоломки, в том числе эти:

сложных загадок, которые растянут ваш мозг | Сложные вопросы

Если вы искали сложные загадки с ответами, вы попали в нужное место! Здравый смысл и опыт не помогут вам ответить на эти каверзные вопросы!

Загадки, которые заставят вас задуматься

Хитрые загадки, которые можно сложить вместе с помощью логики, — отличный способ бросить вызов своему разуму.Думаете, вы сможете решить все из них, не разбираясь в ответах?

1.Комнатная загадка

В какой комнате нет дверей?

|

|

ОТВЕТ

Гриб.

2. Пятеро детей

Разгадай мне эту загадку…

У Пенни пятеро детей:

Первого ребенка зовут Январь.

Второго ребенка зовут Февраль.

Третьего ребенка зовут Март.

Четвертого ребенка зовут Апрель.

Как зовут 5-го малыша.

|

|

Ответ

Сама загадка говорит, что пятого ребенка зовут What

3. новая дверь

Как можно переставить буквы в словах «новая дверь», чтобы получилось одно слово?

|

|

Ответ

«Одно слово».

Загадки с ответами

5. Мальчик и врач

Мальчик и врач рыбачили.

Мальчик был сыном врача, но врач не был отцом мальчика.

Кто доктор?

|

|

Ответ

Его мать.

6. темная комната

Вы входите в темную комнату.

У вас только одно совпадение.

В комнате есть масляная лампа, топка и печь.

Что бы вы зажгли первым?

|

|

Ответ

Сначала зажгите спичку

Загадки, которые заставят вас задуматься

7.Что движется быстрее загадка

Что движется быстрее?

Жара или холод?

|

|

Ответ

Тепло. Потому что всегда можно простудиться.

8. половина пальцев

Каким словом вы бы описали человека, у которого не все пальцы на одной руке?

Ответ

Нормальный , потому что у людей половина пальцев на одной руке.

9. все черное

Мужчина в черном идет по проселочной дороге.

Внезапно из-за угла появляется большая черная машина без света и останавливается.

Как водитель автомобиля узнал, что он там?

Ответ

Потому что было дневное время.

10.Сегодня

Завтра не среда и не четверг .

Вчера не было пятницы или субботы.

Сегодня не четверг , не понедельник и не S унд.

Какой сегодня день?

Ответ

Пятница.

11. Один глаз

Что имеет один глаз, но не видит?

Ответ

Игла.

12. Загадка столь хрупкая

Что такое хрупкое, что его ломает слово?

ОТВЕТ

Тишина .

Теперь попробуйте отгадать 30 сложных загадок!

50 невероятно сложных загадок, которые заставят отгадывать самые острые дети

Если забавная загадка похожа на более сложную версию банальной шутки, то сложная загадка похожа на сложную головоломку, которая в конце награждает детей смехом.Помимо обучения наших детей сочувствию, сотрудничеству, терпению и тому подобному, мы также хотим задействовать их интеллект. Но это не значит, что за обеденным столом им нужно проверять словарный запас или полагаться исключительно на глупые шутки для смеха. Книги, задания и головоломки — все это забавные (акцент на fun ) способы бросить вызов детям и побудить их решать проблемы. Но вы не хотите оскорблять интеллект вашего ребенка, задавая ему головоломки, которые слишком легко решить. По мере того, как они становятся старше, вам придется усерднее работать, чтобы держать их в тонусе, разгадывая все более сложные загадки.

Обдумывая трудную загадку, которая, как вы уверены, поставит вашего ребенка в тупик, вы также должны подумать о том, как работает его мозг. Самая лучшая часть действительно сложных загадок заключается в том, что они награждают каждого ребенка, в зависимости от загадки, которую вы им даете: есть игры с числами для математических ботаников, игра слов для любителей книг и абсурдные головоломки для творческих мыслителей. Ниже приведены самые сложные загадки с ответами, которые ваш ребенок сможет разгадать самостоятельно… проявив достаточно терпения.

  1. Загадка: Какие две вещи нельзя есть на завтрак?
    Ответ: Обед и ужин
  2. Загадка: Что такого хрупкого, что произнесение его имени сломает его?
    Ответ: Тишина.
  3. Загадка: Кто может бегать, но никогда не ходит, имеет рот, но никогда не разговаривает, имеет голову, но никогда не плачет, имеет кровать, но никогда не спит?
    Ответ: Река
  4. Загадка: Что может заполнить комнату, но не занимает места?
    Ответ: Свет
  5. Загадка: Если ты уронишь меня, я обязательно сломаюсь, но улыбнись мне, и я всегда улыбаюсь в ответ.Что я?
    Ответ: Зеркало
  6. Загадка: Какая столица во Франции?
    Ответ : Буква F — единственная заглавная буква во Франции.
  7. Загадка: Чем больше возьмешь, тем больше оставишь. Кто они такие?
    Ответ: Шаги
  8. Загадка: Я поверну один раз, то, что вышло, не войдет. Я поверну снова, то, что есть, не выйдет. Что я?
    Ответ : ключ
  9. Загадка: Мужчина зовет свою собаку с противоположного берега реки.Собака переходит реку, не промокнув, не используя мост или лодку. Как?
    Ответ: Река замерзла.
  10. Загадка: Что ломается, но никогда не падает, и что падает, но никогда не ломается?
    Ответ: День и ночь
  11. Загадка: Что проходит через города и поля, но не движется?
    Ответ: Дорога
  12. Загадка: Я всегда голоден и умру, если не накормить, но все, к чему я прикоснусь, скоро станет красным.Что я?
    Ответ: Пожар
  13. Загадка: Тот, кто его делает, в этом не нуждается; тому, кто его покупает, это не нужно. Человек, который его использует, не может его ни видеть, ни чувствовать. Что это?
    Ответ: Гроб
  14. Загадка: Мужчина смотрит на картину в музее и говорит: «Братья и сестры, у меня их нет, но отец этого человека — сын моего отца». Кто на картине?
    Ответ: Сын мужчины
  15. Загадка: С острыми клыками сижу и жду; с пронзительной силой хрущу судьбу; хватание жертв, провозглашение мощи; физическое соединение одним укусом.Что я?
    Ответ: Степлер
  16. Загадка: У меня есть озера без воды, горы без камня и города без построек. Что я?
    Ответ: Карта
  17. Загадка: Что человек любит больше жизни, ненавидит больше смерти или смертельной розни; то, чего желают довольные люди; бедные имеют, богатые нуждаются; скупец тратит, расточитель копит, а всех несут в могилу?
    Ответ: Ничего
  18. Загадка : Мистери у г-жи Мастард шесть дочерей, и у каждой дочери есть брат. Сколько человек в семье Горчичных?
    Ответ : В семье девять горчиц. Поскольку у каждой дочери один и тот же брат, у нас шесть девочек, один мальчик, а также мистер и мисс Мастард.
  19. Загадка : Меня любят или ненавидят люди. Я меняю внешность и мысли людей. Если человек позаботится о себе, я поднимусь еще выше. Некоторые люди могут захотеть попытаться спрятать меня, но я покажу.Как бы люди ни старались, я никогда не упаду. Что я?
    Ответ : Возраст
  20. Загадка : Только одного цвета, но не одного размера,
    Застрял внизу, но легко летает.
    Присутствует на солнце, но не под дождем.
    Не причиняет вреда и не чувствует боли.
    Что это?
    Ответ : Тень
  21. Загадка : Кто это с шеей и без головы, с двумя руками и без рук?
    Ответ : Рубашка
  22. Загадка : Если одиннадцать плюс два равняется одному, чему равны девять плюс пять?
    Ответ :
    11 часов плюс 2 часа = 1 час
    9 часов плюс 5 часов = 2 часа
  23. Загадка : Можете ли вы записать восемь восьмерок, чтобы они в сумме давали тысячу?
    Ответ : 888 + 88 + 8 + 8 + 8 = 1000
  24. Загадка : Его нельзя увидеть, нельзя почувствовать, нельзя услышать и нельзя понюхать.Он лежит за звездами и под холмами, И пустые дыры он заполняет. Оно приходит первым и следует после, Конец жизни и убивает смех. Что это?
    Ответ : Тьма
  25. Загадка: Какое английское слово сохраняет то же произношение даже после того, как вы уберете четыре из пяти букв?
    Ответ: Очередь
  26. Загадка: Три игральные карты подряд. Можете ли вы назвать их этими подсказками? Справа от короля стоит двое. Слева от лопаты находится бриллиант.Туз слева от сердца. Сердце слева от лопаты. Теперь определите все три карты.
    Ответ: Три игральные карты подряд. Можете ли вы назвать их этими подсказками? Справа от короля стоит двое. Слева от лопаты находится бриллиант. Туз слева от сердца. Сердце слева от лопаты. Теперь определите все три карты.
    Ответ: Бубновый туз, Червовый Король, Пиковая двойка.
  27. Загадка: Что такое, если дать одну, у вас будет либо две, либо ни одной?
    Ответ: Выбор.
  28. Загадка: Джордж, Хелен и Стив пьют кофе. Берт, Карен и Дэйв пьют газировку. Используя логику, Элизабет пьет кофе или газировку?
    Ответ: Элизабет пьет кофе. Буква E встречается дважды в ее имени, как и в именах других, пьющих кофе.
  29. Загадка: У меня есть ветви, но нет листьев, ствола и фруктов. Что я?
    Ответ: Банк.
  30. Загадка: У чего голова, хвост, коричневый, а ног нет?
    Ответ: Пенни.
  31. Загадка: Когда меня используют, я бесполезен. Однажды предложили, вскоре отвергли. Я часто не выражаю отчаяния. Что я?
    Ответ: Предлог.
  32. Загадка: У меня четыре ноги, но нет волос. Люди катаются на мне часами, но никуда не уезжают. Мне всегда удается быть готовым к работе, без необходимости дергать или включать. Что я?
    Ответ: Стол
  33. Загадка: Какое из следующих слов не входит в группу и почему? CORSET, COSTER, SECTOR, ESCORT, COURTS
    Ответ: Суды.Все остальные являются анаграммами друг друга.
  34. Загадка: Когда Джону было шесть лет, он забил гвоздь в свое любимое дерево, чтобы отметить свой рост. Десять лет спустя, в возрасте шестнадцати лет, Джон вернулся, чтобы посмотреть, насколько выше был гвоздь. Если бы дерево росло на пять сантиметров каждый год, насколько выше был бы гвоздь?
    Ответ: Гвоздь будет на той же высоте, поскольку деревья растут на их вершинах.
  35. Загадка: Я в четыре раза старше своей дочери.Через 20 лет я буду вдвое старше нее. Сколько нам сейчас лет?
    Ответ: Мне 40 лет, дочери 10 лет.
  36. Загадка: Был человек, который родился раньше своего отца, убил его мать и женился на его сестре. Тем не менее, в том, что он сделал, не было ничего плохого. Почему?
    Ответ: Его отец был впереди него, когда он родился, поэтому он родился раньше него. Его мать умерла при родах. Наконец, он вырос и стал священником и женился на своей сестре на ее церемонии.
  37. Загадка: Лилия — кувшинок в небольшом пруду. Лили удваивает размер каждый день. На 20-й день она покрывает весь пруд. В какой день Лили была размером с половину пруда?
    Ответ: День 19, это не 10, потому что на 20-й день она увеличилась вдвое по сравнению с 19-м днем, поэтому 19 должен быть половиной размера пруда.
  38. Загадка: Какой распространенный английский глагол становится своим прошедшим временем, переставляя свои буквы?
    Ответ: Ешь и ешь.
  39. Загадка: На вопрос, сколько ей лет, Сьюзи ответила: «Через два года я буду вдвое старше, чем была пять лет назад.» Сколько ей лет?
    Ответ: Ей 12.
  40. Загадка: Легче, чем то, из чего я сделан, Больше меня скрыто, Чем видно. Что я?
    Ответ: Айсберг.
  41. Загадка: Она короче остальных, но когда вы счастливы, вы поднимаете ее, как лучшую. Что это?
    Ответ: Большой палец.
  42. Загадка: Человек, купивший ее, не хочет ее себе. Человек, который его покупает, покупает это не для себя.И человек, которому это нужно, не знает, что это нужно ему. Что это?
    Ответ: Гроб.
  43. Загадка: У меня есть города, но нет домов. У меня есть горы, но нет деревьев. У меня есть вода, но нет рыбы. Что я?
    Ответ: Карта.
  44. Загадка: У девушки столько братьев, сколько сестер, но у каждого брата братьев вдвое меньше, чем сестер. Сколько братьев и сестер в семье?
    Ответ: Четыре сестры и три брата.
  45. Загадка: Сначала подумайте о цвете облаков. Затем подумайте о цвете снега. А теперь подумайте о цвете яркой полной луны. А теперь быстро ответьте: что пьют коровы?
    Ответ: Вода
  46. Загадка: Разные огни делают меня странным, поэтому я буду менять их на разные размеры. Что я
    Ответ : Я зрачок глаза.
  47. Загадка: У чего есть хвост и голова, но нет тела?
    Ответ : Монета
  48. Загадка: Я нечетное число.Забери письмо и я стану ровным. Какой я номер?
    Ответ: Семь
  49. Загадка: Если красный дом сделан из красного кирпича, а желтый — из желтого кирпича, из чего сделана теплица?
    Ответ: Все теплицы стеклянные.
  50. Загадка: Что нельзя класть в кастрюлю?
    Ответ: Крышка.

Ой! Пожалуйста, попробуйте еще раз.

Спасибо за подписку!

21 лучшая загадка — вопросы и ответы

Разгадайте эти сложные загадки, чтобы увидеть, сможете ли вы противостоять вызову.Здесь мы собрали для вас 21 каверзную загадку с ответами. Мы собрали воедино некоторые из самых сложных и простых загадок со всего Интернета, чтобы увидеть, сможете ли вы противостоять психологическому испытанию. Вы можете поделиться этими загадками с членами семьи, друзьями и посмотреть, смогут ли они их разгадать.

Вы также можете попробовать несколько забавных загадок . Эти загадки одновременно забавны и интересны.

Или попробуйте эту сложную головоломку, где вам предлагается найти спрятанного кота. Я уверен, что найти эту спрятанную кошку будет сложно, но ты можешь ее найти.

  1. X — отец Y. Но Y не является сыном X. Как такое возможно? (Ответ: Y — дочь X )

  2. Мужчина во всем черном идет по проселочной дороге. Внезапно из-за угла появляется большая черная машина без света и с визгом останавливается. Как машина узнала, что он там? (Ответ: было днем. )

  3. Петух отложил яйцо на крыше сарая.В какую сторону он катился? (Ответ: Не катился — с каких это пор петухи начали откладывать яйца? )

  4. Водитель грузовика едет по улице с односторонним движением не в том направлении и проезжает мимо как минимум десяти полицейских. Почему его не поймали? (Ответ: Потому что он не вел машину! Он идет по тротуару. )

  5. Электропоезд движется на север со скоростью 100 миль в час, а ветер дует с запада со скоростью 10 миль в час. Куда дует дым? (Ответ: С электричкой дыма нет.)

  6. Как может человек восемь дней не спать? (Ответ: Ночью )

  7. Как можно уронить сырое яйцо на бетонный пол, не треснув его? (Ответ: Яйцо не треснет бетонный пол! )

  8. Если врач даст вам 3 таблетки и скажет принимать одну таблетку каждые полчаса, сколько времени пройдет, прежде чем будут приняты все таблетки? (Ответ: 1 час! Примите сразу 1-ю таблетку, через полчаса примите 2-ю, а через полчаса — 3-ю.Общее затраченное время: 1 час! )

  9. Если павлин мистера Смита откладывает яйцо во дворе мистера Джонса, кому принадлежит яйцо? (Ответ: Павлины не откладывают яиц, а только павлины. )

  10. Если есть 6 яблок, и вы убираете 4, сколько у вас есть? (Ответ: Число взятых вами 4. )

  11. Если бы у вас была только одна спичка, и вы вошли в темную комнату с масляной лампой, газетой и дровами, что бы вы зажгли в первую очередь? (Ответ: Матч.)

  12. В некоторых месяцах 31 день, в других 30 дней. Сколько у 28 дней? (Ответ: Во всех месяцах 28 дней. )

  13. У отца Марии пять дочерей по имени Мала, Мили, Меле, Моло… Угадайте, как назовут пятую? (Ответ: Мэри! Она была бы пятой дочерью )

  14. Что идет вверх и вниз, но остается на том же месте? (Ответ: Лестница! )

  15. Как далеко вы можете пройти в лес? (Ответ: На полпути.После этого вы выходите из леса. )

  16. Что произойдет, если вы бросите синий камень в желтое море? (Ответ: станет мокрым. )

  17. Что становится влажнее и влажнее, чем больше сохнет? (Ответ: Полотенце )

  18. Что идет вверх и никогда не падает? (Ответ: Возраст !)

  19. Что тяжелее: 100 фунтов камней или 100 фунтов перьев? (Ответ: Они оба весят одинаково — 100 фунтов )

  20. У чего есть голова и хвост, но нет тела? (Ответ: Монета )

  21. Сколько раз алфавит «а» встречается от 0 до 100.(Ответ: Нет ).

Комментарии

комментария

Прикольные головоломки, Пазлы для детей, Загадки, Простые головоломки, Загадки для WhatsApp Отвечайте на хорошие загадки, чтобы переслать их друзьям, Отвечайте на экзамен IAS, Отвечайте на головоломки WhatsApp, ответы на загадки WhatsApp, игра для мозга, Головоломки, головоломки для тренировки мозга, вопросы CAT, сложные вопросы CAT, сложные вопросы экзамена CET, Бросьте вызов своему мозгу, завершите слова, запутанные загадки для WhatsApp, поиск слов, забавные головоломки для мозга, веселые викторины, забавные загадки, общие знания, головоломки с общими знаниями, ответы на загадки с общими знаниями, загадки с общими знаниями, хорошие загадки для WhatsApp, сложные головоломки, вопросы IAS , Iine вперед и загадки, головоломки, головоломки ответы на вопросы интервью, головоломки из картинок, головоломки и интеллектуальные игры, головоломки игры, Riddle, ответы на загадки, головоломки головоломки, веб-сайты с загадками, загадки, загадки и интеллектуальные игры, загадки и головоломки, загадки для watsapp, загадки для WhatsApp, загадки загадки ответы картинки, sms-переадресация решения для головоломки WhatsApp, разгадывать загадку отсутствующих гласных, решить это, если вы гений, решить это я Если вы гений загадки, загадки воскресного дня, загадки воскресенья, сложные вопросы CAT, сложные вопросы IIM, сложные вопросы интервью IIT, сложные вопросы собеседования, загадка с трудным числом, сложные загадки с ответами, загадка следующего номера WhatsApp, загадки WhatsApp и сообщения, Загадки WhatsApp с ответами, сложная головоломка WhatsApp, словесные забавные вопросы, словесные загадки ответы

О Кумаре

Привет всем, я Рамеш Кумар, автор этого блога.Я ИТ-инженер. Я люблю головоломки и загадки, поэтому решил опубликовать их и для других, как я. Если вам нравятся головоломки, подпишитесь на мою рассылку. Вы также можете следить за мной через страницу сайта Facebook. Если вы хотите связаться со мной, воспользуйтесь формой на странице контактов, я отвечу на ваши электронные письма как можно скорее. Спасибо

29 сложных загадок с ответами — сложные загадки

Эти каверзных загадок с ответами согнут ваш мозг! Проверьте себя и посмотрите, сколько из этих обманчиво сложных загадок вы сможете разгадать.Затем бросьте вызов своим друзьям, семье и коллегам и посмотрите, насколько хорошо они могут это сделать. Посмотрите внимательно и попытайтесь подойти к ним непредвзято, потому что вы уже знаете, что они созданы для того, чтобы подшутить над вами. Не заблуждайтесь, это определенно непростые загадки, но они, безусловно, будут веселыми и интересными. Некоторые из них забавные, некоторые глупые, некоторые длинные, а третьи — короткие сложные загадки. В дополнение к приведенной ниже коллекции из сложных загадок и ответов , мы также включили специальный раздел из сложных загадок для детей .Будьте готовы к тому, что ваш мозг дразнят!

Загадки с ответами

Как сложить восемь шестерок, чтобы в сумме получилось 750?

Показать ответ

666 + 66 + 6 + 6 + 6 = 750

Маленькая девочка идет в магазин и покупает дюжину яиц, а когда она идет домой, все, кроме трех, разбиваются.Сколько яиц осталось целыми?

Пятница, Джейсон и Джефф пошли в магазин, чтобы купить хлеба. Сумма 12 долларов была разделена поровну между друзьями. Джейсон заплатил 4 доллара, и Джефф тоже заплатил 4 доллара. Кто заплатил остальные 4 доллара?

Показать ответ

Их друг Пятница

В одноэтажном доме на углу дороги спальни были желтыми, кухня — синей, гостиная — оранжевой, гараж — зеленым, холл — красным, а гостиная — фиолетовым.Какого цвета была лестница?

Показать ответ

В одноэтажном доме нет лестниц

Люди заставляют меня держать меня, меняют, воспитывают, даже если я могу быть очень грязным. Что я?

Сможете ли вы решить эти действительно сложные математические загадки?

Бабушка, две матери и две дочери пошли в кафе попить чаю.Они заказали по одной чашке каждому. Какое общее количество чашек они заказали?

Показать ответ

3, по одному на каждого человека (бабушка — тоже мать, а мать — тоже дочь) — ИЛИ — 2 (оба человека — матери и дочери, и одна — бабушка)

Франк родился в 1933 году и умер в 1946 году в возрасте 57 лет.Как это может быть?

Показать ответ

Родился в 1933 г. в больничной палате, умер в 1946 г.

Что из перечисленного не является белым? Яичные белки, Белый дом, белые медведи, мячи для гольфа, молоко, зефир и белый лук.

Показать ответ

Белые медведи, потому что у них бесцветный мех.Каждая прядь меха на самом деле прозрачна и не содержит пигментов с полой сердцевиной, которая отражает свет.

После 8 часов полета из аэропорта Лос-Анджелеса в аэропорт Манилы, в каком направлении вы бы приземлились в Маниле?

Показать ответ

Нет, потому что вы все еще будете где-то за океаном, потому что обычно дорога из Лос-Анджелеса в Манилу занимает около 13 часов.

Какая река была самой длинной в мире до открытия реки Амазонки?

Показать ответ

Амазонка, потому что она была самой длинной даже до того, как ее обнаружили

Я несколько дюймов в длину и пару дюймов в ширину, и большинству женщин хотелось бы схватить меня.Что я?

Показать ответ

Слиток золота

Сумасшедший ученый хотел взять человека, у которого было 5 голов, удалить 4 из них и пришить их к голове клоуна. По его оценкам, это займет 43 минуты на человека. Сколько времени ему потребуется, чтобы завершить всю процедуру?

Показать ответ

Он не может завершить это, потому что ни у кого нет 5 голов

Иванка курит электронную сигарету и дует ей на Джареда, и это его беспокоит, и она следует за ним, куда бы он ни пошел.Что мог сделать Джаред, чтобы избежать дыма?

Показать ответ

Ничего, так как электронные сигареты не дымят

Ни один самолет не приземлился в аэропорту Цинциннати. Как это может быть?

Показать ответ

Потому что на самом деле в Цинциннати нет аэропорта.Ближайший аэропорт находится через границу штата в Кентукки.

Если вы покрасите коричневый дом в белый цвет, он станет белым домом, если сигнал стоп-сигнала изменится с красного на зеленый, то он станет зеленым. Итак, если вы бросите белую рубашку в Красное море, чем она станет?

Показать ответ

Намокнет. Цвета не изменится, потому что Красное море на самом деле не красное

Если индейки — на День благодарения, жокеи — на лошадей, а обезьяны — на джунгли, то что для быдлов?

Если тигр-самец — тигрице, петух — курице, а бык — корове, тогда самец — чему?

Показать ответ

Очень грубая девушка (если вы этого не понимаете, возможно, вы слишком молоды)

Сколько дней рождения у среднего Свидетеля Иеговы?

Показать ответ

Нет, дни рождения не празднуют

Если в военном лагере 183 солдата и у военного повстанца есть один выстрел, чтобы убить их всех, что бы повстанец сделал, чтобы убедиться, что один выстрел поразил всех 183?

Показать ответ

Он просил всех 183 встать поближе, чтобы все поместились на фото

Детские загадки с ответами

Все любят вызовы, и, более того, мы любим, когда мы можем принять вызов и приложить все усилия.Эти замысловатые загадки для детей помогут вам научить своих детей и учеников мыслить творчески и внимательно наблюдать. Тренировка детского мозга чем-то вроде сложных загадок может помочь воспитать острый и проницательный ум. В то время как некоторые из очень сложных загадок сверху могут быть сочтены некоторыми сложными загадками только для взрослых, загадки здесь были специально созданы, чтобы больше подходить для детей. Также обратите внимание на наш самый большой выбор загадок для детей.

Если дорога из желтого кирпича — желтая, синяя из Blue’s Clues — синяя, а Белый дом — белый, то какого цвета теплица?

Показать ответ

Прозрачный, т.к. сделан из стекла

В ванне плавали 4 резиновых утки, 2 уплыли и 2 утонули.Сколько еще живых резиновых уток?

Показать ответ

Ноль, потому что резиновые утки не живы и не могут утонуть

Пара обуви и пара носков стоят в общей сложности 1,10 доллара. Обувь стоит на 1 доллар дороже носков. Сколько стоят носки?

Показать ответ

$ 0.05 (не 0,10 доллара США)

Если вы купите петуха, чтобы вы могли собирать и есть яйца каждое утро, и ожидаете, что каждое утро на завтрак будете есть три яйца, то сколько яиц у вас будет через 10 дней?

Показать ответ

Ноль, потому что петухи не откладывают яиц

Что движется вверх и вниз и кружится с невероятно высокой скоростью?

Показать ответ

Земля, солнце и большинство вещей во вселенной

Что из следующего верно: 1) Страус умеет летать или 2) Страусы умеют летать?

Показать ответ

Ни то, ни другое, потому что страусы не умеют летать

Если E + 1 = 4, D + 1 = 3, E — D = 1, то что вы получите, если сложите E и 3 вместе?

Показать ответ

Что-то похожее на цифру 8 (если сложить их рядом)

Кто имеет руки, но не может носить перчатки, лицо, но не может смотреть?

Что самое маленькое: метр, 3.

Добавить комментарий

Ваш адрес email не будет опубликован. Обязательные поля помечены *